المساعد الشخصي الرقمي

مشاهدة النسخة كاملة : التماثل و التماثل الفائق



الصادق
08-09-2009, 01:21 AM
التماثُل و التماثُل الفائق


مدخل:

ماذا نعنى بمفهوم التماثل او التناظر؟

التماثل او التناظر بوجه عام هو احد الخصائص الجمالية للاشكال الهندسية و النظريات الفيزيائية, وتفادياً للمسألة الذوقية فاننا نعنى بالجمال هنا البساطة. فعندما يصف الفيزيائى نظرية ما بانها جميلة فهو حتماً يعنى انها نظرية بسيطة متماثلة فى بنيتها الداخلية و قادرة على وصف الطبيعة.كمثال لذلك نجد ان النظرية النسبية الخاصة مبنية على مفهوم التماثل وكون المبداء الاول من مبادئ النظرية النسبية يقول ان جميع مناطات الاسناد القصورية متكافئة (اى متماثلة ومتناظرة) فى وصف الطبيعة فهو يدل على هذا البعد التماثلى الجمالى للنظرية. ثم جاء المبداء الثانى ليشد من عضد المبداء الاول فى هذه السمفونية الجمالية

مثال(1) :

افترض معادلة القطع المكافئ التالية
http://www.codecogs.com/eq.latex?\huge%20y(x)=x^2
الان دعنا نقوم بتحويل المتغير المستقل من x الى سالب x

وهكذا سوف تكون معادلة القطع المكافئ بعد التحويل هى
http://www.codecogs.com/eq.latex?\huge%20y(-x)=(-x)^2=x^2

والان طالما ان التحويل لم يغير المعادلة فاننا نقول ان معادلة القطع المكافئ متماثلة نتيجة للتحويل http://www.codecogs.com/eq.latex?\large%20x\rightarrow%20-x

http://students.umf.maine.edu/~nieuwkhj/reallifequadraticswebquest/parabola%201.gif

نلاحظ من الرسم ان النصفين الايمن والايسر متماثلين.

الصادق
08-09-2009, 02:57 AM
مثال(2):

هب انه لدينا جهاز قياس حرارة (ثيرمومتر) وقمنا بقياس درجة الحرارة عند نقطة x داخل الغرفة ووجدنا ان درجة الحرارة عند تلك النقطة تساوى 20 درجة مئوية ثم بعد ذلك قمنا بقياس درجة الحرارة عند نقطة اخرى x+a داخل الغرفة (اى قمنا بانتقال مكانى (تحويل) من نقطة الى اخرى ) ووجدنا ان درجة الحرارة عند النقطة الجديدة تساوى ايضاً 20 درجة مئوية. وهكذا كانت درجة الحرارة عند جميع نقاط الغرفة تساوى 20 درجة. الان ماهو الاستقراء الفيزيائى الذى سوف نخلص اليه؟

بالطبع سوف نقول ان درجة الحرارة موزعة بانتظام داخل الغرفة اى ان درجة الحرارة لا تتغير نتيجة للانتقال المكانى داخل الغرفة بمعنى اخر ان درجة الحرارة متماثلة عند جميع النقاط

التماثل يقود الى الثبات

وجدنا ان درجة الحرارة عند x تساوى 20 درجة مئوية اى ان

http://www.codecogs.com/eq.latex?\huge%20T(x)=20^o

وعند النقطة x+a كانت درجو الحرارة ايضاً تساوى 20 درجة مئوية اى ان

http://www.codecogs.com/eq.latex?\huge%20T(x+a)=20^o

وهكذا فان

http://www.codecogs.com/eq.latex?\huge%20T(x+a)=T(x)

وبأخذ مفكوك تايلور للدالة فى الطرف الايسر من المعادلة الاخيرة نجد ان

http://www.codecogs.com/eq.latex?\huge%20T(x)+a\frac{dT}{dx }+\frac{a^2}{2!}\frac{d^2T}{dx^2}+. ..=T(x)

وهكذا نجد ان

http://www.codecogs.com/eq.latex?\huge%20a\frac{dT}{dx}+\fr ac{a^2}{2!}\frac{d^2T}{dx^2}+...=0

ولما كانت a قيمة اختيارية (اعتباطية) فانه يكفى ان تساوى المشتقة الاولى صفراً للتحقق المعادلة الاخيرة اى
http://www.codecogs.com/eq.latex?\huge%20\frac{dT}{dx}=0%20 \qquad%20\Rightarrow%20T=constant

اى انه نتيجة لتماثل تحت تحويل الانتقال المكانى فان درجة الحرارة تظل ثابته عند جميع النقاط داخل الغرفة

يتبع .....

NEWTON
08-09-2009, 10:20 PM
السلام عليكم ..

أخي الغالي "الصادق" ..
والله لساني لعاجز .. ولا يحضرني شيء استطيع به أن أشكرك على طرح هذا الموضوع المتميز ، من باقة مواضيعك التي تتحفنا بها دائما ..

أنتظر البقية على أحر من الجمر :) ..

بارك الله فيك وبك.

مروة إبراهيم
08-10-2009, 02:44 AM
بارك الله فيك أستاذ الصادق
وجزاك الله خير ونفع بك

بصراحة أستاذ الصادق..لك قدرة على توصيل المادة العلمية في صورة سهلة وواضحة
ولك أسلوب مميز في الشرح لا يضاهيه أسلوب
ولذلك فأنا كمان أنتظر البقية على أحر من الجمر :) :)

Tyns19
08-10-2009, 08:24 AM
السلام عليكم
أستاذنا الصادق في كل مرة أقرأ أحد مواضيعك أزداد اعجابا بأسلوبك (خاصة موضوع #معادلة انشتاين فى النسبية العامة#).
بارك الله فيك أخي وزادك علما وايمانا.
عندي استفسار صغير:
لتكن الدالة التالية:

http://www.codecogs.com/eq.latex?\150dpi%20\[T \left( x \right) =\cos \left( {\frac {2\pi}{\tau}}x \right) \]

في غالب الأحيان نقول ان http://www.codecogs.com/eq.latex?\150dpi%20\[\tau\] هو دور الدالة
سؤالي هو:
هل للدوال الدورية أي علاقة بهذا الموضوع حيث أنها حالة خاصة من العلاقة:

http://www.codecogs.com/eq.latex?\huge%20T(x+ \tau)=T(x)
حيث في الدوال الدورية http://www.codecogs.com/eq.latex?\150dpi%20\[\tau\] هو عدد ثابت وليس اعتباطي كما هو الحال في مثالك السابق.
أظن في هذه الحال أن هناك عددا لانهائي من التحويلات المكانية ستحافظ على ثبات درجة الحرارة (في المثال المذكور أعلاه)، حيث بحسب تعريف الدوال الدورية:

http://www.codecogs.com/eq.latex?\150dpi%20\ T(x+ \tau)=T(x) \Rightarrow T(x+ n\cdot \tau)=T(x)
(n عدد صحيح)
شكرا.

فريدة
08-11-2009, 02:34 AM
السلام عليكم
بارك الله فيك أستاد و بي اسلوبك المميز كالعادة وزادك علما وجزاك الله خير

بدرسك هاد أرجعتني إلى الوراء دروس الثانوية العامة ..........بتع التحويلات الهندسية
و إدا كان تعريفك للتماثل فما الفرق بينه و بين التماثُل الفائق???
و ما هو الهدف من درس هده التماثُلآت??

تحياتي
الاخت فريدة

الصادق
08-12-2009, 04:19 AM
السلام عليكم

اخى العزيز NEWTON
بارك الله فيك على الكلمات الطيبات و لك الشكر والتقدير و اتمنى ان تجد فى الموضوع ما يرضيك


اختى الكريمة مروة إبراهيم

لك الشكر على كلماتك الطيبات و بارك الله فيك وجزاك خيراً وذادك من فضله. وسوف اوصل فى الموضوع انشاءالله


اخى Tyns19
شكر اً على السؤال الذكى جداً وانت لا تحتاج اجابتى لان اجابتك صحيحة
دور الدالة الدورية هو خاصية لتلك الدالة وهو مقدار ثابت للدالة المعينة و دور دالة الجيب مثلاً هو 360 درجة و انت لا تستطيع ان تميز بين جيب الزاوية x و جيب الزاوية x+360 ولكنك حتماً تستطيع ان التميز جيب الزاوية x عن جيب الزاوية x+359
اما التحويل المكانى فهو اعتباطى فمثلاً اذا كانت x لا متغيرة تحت عملية التحويل الانتقالى فانت لا تستطيع ان تميز بين x و x+1 و x-0.765 و x+1000 بمعنى انه لاتوجد قيمة ثابته لمقدار التغير


الاخت فريدة
بارك الله فيك و جزاك كل خير
معك حق فى انها رياضيات الثانوية العامة و لكنى لم اتحدث عن التماثل فى الفيزياء بعد والمثال (1) هو كان فقط من اجل تعريف كلمة تماثل و كنت اريد ان اعطى القارئ احساس ان التماثل يتتطلب و جود تحويل ما. و اجابة السؤاليين اعلاه سوف تفصح عنهما المشاركات القادمات انشاء الله و اتمنى ان تتابعى معنا الموضوع حتى النهاية

الصادق
08-12-2009, 07:14 AM
تماثلات الاحداثيات

دعنا اولاً نُعرف نظام الاحداثيات ومن اجل هذا الغرض دعنا نعتبر المثال التالى:

مثال (3): افترض انه لدينا ذرات غاز موزعة فى الفضاء ومن اجل التبسيط افترض ان هذه الذرات تعيش فى فضاء به بعد واحد هو x وليكن احداثى الذرة رقم 1 هو http://latex.codecogs.com/gif.latex?\large%20x_1 و احداثى الذرة رقم 2 هو http://latex.codecogs.com/gif.latex?\large%20x_2 و هكذا فان احداثى الذرة رقم i هو http://latex.codecogs.com/gif.latex?\large%20x_i. و الان اذا اخذنا فى الاعتبار التحويل الانتقالى فاننا نجد ان
http://latex.codecogs.com/gif.latex?\huge%20x_i\to%20x_i%27=x _i+\delta%20x

حيث ان http://latex.codecogs.com/gif.latex?\large%20\delta%20x تمثل قيمة اعتباطية وهى بالطبع لا تعتمد على الذرة المعينة لذلك لا يظهرفيها المعامل i , ولذلك فاننا لا نستطيع ان نقيس الموضع http://latex.codecogs.com/gif.latex?\large%20x_i لاننا لا نستطيع ان نميز بين http://latex.codecogs.com/gif.latex?\large%20x_i و http://latex.codecogs.com/gif.latex?\large%20x_i+\delta x

الدرس الاول : لا يمكن قياس موقع نقطة فى الفضاء...............

الان دعنا نتحدث عن الذرة رقم i عند الموضع http://latex.codecogs.com/gif.latex?\large%20x_i والذرة رقم j التى عند الموضع http://latex.codecogs.com/gif.latex?\large%20x_j

نُعرف البعد بين الذرة رقم i والذرة رقم j ب

http://latex.codecogs.com/gif.latex?\huge%20x_{ij}=x_i-x_j%20\qquad(1)

باجراء التحويل الانتقالى نجد ان

http://latex.codecogs.com/gif.latex?\huge%20\\x_i%27=x_i+\del ta%20x%20\qquad(2)\\%20\\%20x_j%27= x_j+\delta%20x%20\qquad(3)

الان بطرح المعادلة (3) من المعادلة (2)

http://latex.codecogs.com/gif.latex?\huge%20x_i%27-x_j%27=(x_i+\delta%20x)-(x_j+\delta%20x)=x_i-x_j
وهكذا فان البعد بين النقطتين لا يتغير نتيجة للتحويل الانتقالى
http://latex.codecogs.com/gif.latex?\huge%20x_{ij}%27=x_{ij}
وهكذا طالما ان البعد بين الذرتين i و j لا يتغير نتيجة للتحويل الانتقالى فاننا نستطيع قياس البعد بينهما
الدرس الثانى: لا نستطيع قياس موقع نقطة فى الفضاء و لكنا نستطيع قياس البعد بين نقطتين.....

دائماً تضع التماثلات شروط يجب تحقيقها. وفى حالتنا هذه فانه و اضح من العلاقة (3) ان الازحة من النقطة i النقطة j تساوى سالب الازاحة من j الى i
http://latex.codecogs.com/gif.latex?\huge%20x_{ji}=-x_{ij}%20\qquad(4)

اما العلاقة الثانية فهى ان الازاحة من النقطة i الى النقطة j ومن النقطة j الى النقطة k ومن k الى النقطة i مرة اخرى تساوى صفراُ اى ان

http://latex.codecogs.com/gif.latex?\huge%20x_{ij}+x_{jk}+x_{ ki}=0%20\qquad(5)

سوف نسمى الشرط فى المعادلة (4) بعلاقة ضد التماثلية اما الشرط فى المعادلة (5) بعلاقة التدوير

حيلة قديمة:
يمكن ان نحدد نقطة محددة و تكن تلك النقطة هى موقع الذرة رقم 1 و لنسمى هذه النقطة بنقطة الاصل
http://latex.codecogs.com/gif.latex?\huge%20x_1=0

وهكذا فان البعد بين النقطة i و النقطة 1 هو
http://latex.codecogs.com/gif.latex?\huge%20x_{i1}=x_i-x_1=x_i-0=x_i

الدرس الثالث: عند تعريف نقطة ما على انها نقطة الاصل فان http://latex.codecogs.com/gif.latex?\large%20x_i اصبح يمثل بعد النقطة i من نقطة الاصل
وعند اجراء تحويل الانتقال فاننا نحصل على
http://latex.codecogs.com/gif.latex?\huge%20x_{i1}%27=x_i%27-x_1%27=(x_i+\delta%20x)-(0+\delta%20x)=x_i
اى انه بعد تعريف نقطة الاصل فان http://latex.codecogs.com/gif.latex?\large%20x_i لا تتغير نتيجة للتحويل الانتقالى بعكس ماهو الحال قبل تعريف نقطة الاصل.

تمرين:
1-برهن العلاقات (4) و (5) مستخدماً العلاقة (3)
2- فى الفيزياء عادة ما نقوم باختيار نفطة اصل الاحداثيات و نريح انفسنا تماماً من قضية التماثل نتيجة تحويل الانتقالى. هل تستطيع ان تزكر حالة فضاء فى الفيزياء لا نقوم فيه بتعريف نقطة الاصل
3- عمم المثال السابق الى حالة غاز من الذرات فى فضاء ثلاثى الابعاد


يتبع.............

Tyns19
08-12-2009, 08:08 PM
السلام عليكم
الشكر لك الأخ الصادق لقد كفيت و وفيت
هذا هو حل التمرين ان شاء الله:
1-برهن العلاقات (4) و (5) مستخدماً العلاقة (3):
باستخدام العلاقة (3) نجد:


http://www.codecogs.com/eq.latex?\150dpi%20 \left.\begin{matrix}x_{ji}=x_j-x_i\\ x_{ij}=x_i-x_j\end{matrix}\right\}\Rightarrow x_{ji}=-x_{ij}

http://www.codecogs.com/eq.latex?\150dpi%20\ x_{ij}+x_{jk}+x_{ki}=(x_i-x_j)+(x_j-x_k)+(x_k-x_i)=0

2- فى الفيزياء عادة ما نقوم باختيار نفطة اصل الاحداثيات و نريح انفسنا تماماً من قضية التماثل نتيجة تحويل الانتقالى. هل تستطيع ان تزكر حالة فضاء فى الفيزياء لا نقوم فيه بتعريف نقطة الاصل

بالي مشغول بعض الشيء هذه الأيام لكن كأول تخمين خطر في بالي الفضاء الكهروستاتيكي حيث يكتب قانون كولوم مثلا وبالنسبة إلى شحنتين كالتالي:


http://www.codecogs.com/eq.latex?\150dpi%20 \vec{F}(\vec{r})=\frac{1}{4\pi\vare psilon_0}\frac{q_1q_2}{|\vec{r}_1-\vec{r}_2|^3}(\vec{r}_1-\vec{r}_2)
أما في حالة N شحنة فان القوة الكلية التي تؤثر بها كل الشحنات على الشحنة رقم i هي:


http://www.codecogs.com/eq.latex?\150dpi%20 \vec{F}_i(\vec{r})=\frac{q_i}{4\pi\ varepsilon_0}\sum_{j=1\above{j\neq i}}^N\frac{q_j}{|\vec{r}_i-\vec{r}_j|^3}(\vec{r}_i-\vec{r}_j)

3- عمم المثال السابق الى حالة غاز من الذرات فى فضاء ثلاثى الابعاد


http://www.codecogs.com/eq.latex?\150dpi%20 \vec{r}_i\rightarrow\vec{r’}_i=\vec {r}_i+\delta\vec{r}\\\vec{r}_{ij}=\ vec{r}_i-\vec{r}_j\qquad(1).\\\vec{r}_{ij}+\ vec{r}_{jk}+\vec{r}_{ki}=\vec{0}\qq uad (5).

لاحظ أن العلاقة (4) هي حالة خاصة من العلاقة (5). بقي فقط أن أشير أن التحويل سيكون صالح تحت جميع نظم الاحداثيات وهذا ما تعودناه طبعا في قوانين الفيزياء حيث مثلا في الاحداثيات الكرتيزية:


http://www.codecogs.com/eq.latex?\150dpi%20 \vec{r}_n=x_n \vec{i}+y_n\vec{j}+z_n\vec{k}\\\del ta\vec{r}=\delta x\cdot \vec{i}+\delta y\cdot\vec{j}+\delta z\cdot\vec{k}

شكرا أتمنى أن أكون قد وفقت ولو قليلا.
ملاحظة:
إذا كان الموضوع لا يحتمل حل التمارين في نفس الصفحة فإن ليس لدي أي مانع في حذف هذه المشاركة أو نقلها.

في انتظار باقي الموضوع انشاء الله.......

الصادق
08-14-2009, 04:26 AM
اخى Tyns19
اشكرك فقد اسعدتنى جداً بحلك للتمرين

و اجابتك للسؤاليين الاول و الثالث صحيحة تماماً

ولكن هناك مشكلة فى حل السؤال الثانى لان الحالة التى طرحتها هى حالة شحنات موضوعة فى نقاط غير نقطة الاصل و لكن هذا لا يمنع من وجود نقطة الاصل فهى معرفة ب http://latex.codecogs.com/gif.latex?\huge%20\vec{r}=0

كنت اقصد بسؤالى فضاء (وليس نظام احداثيات) و ليس بالضرورة ان يكون فضاء مكانى او زمنكانى

اشكرك اخى Tyns19 على الحلول الرائعة هذه وانتظر منك محاولة اخرى للسؤال الثانى

Tyns19
08-14-2009, 09:25 PM
اخى tyns19
اشكرك فقد اسعدتنى جداً بحلك للتمرين
الشكر لك أستاذي على طرحك لمواضيع متميزة، أما نحن فأقل ما يمكن أن نفعله هو تشجيعك على المواصلة وأنا أؤمن أن التشجيع يكون عن طريق التفاعل والمشاركات البناءة(والكلام طويل في هذا السياق...).

كنت اقصد بسؤالى فضاء (وليس نظام احداثيات) و ليس بالضرورة ان يكون فضاء مكانى او زمنكانى

اشكرك اخى tyns19 على الحلول الرائعة هذه وانتظر منك محاولة اخرى للسؤال الثانى
سبق و أن أشرت في مشاركة سابقة أنني أعاني حالة تشتت، و كلامك هذا أدخلني في أمور فلسفية وفيزيائية عديدة ومع هذا فلم أخرج بنتيجة تذكر أتمنى أن تعطينا الحل حتى نستطيع مواصلة الموضوع كما أرجو من الإخوة أعضاء المنتدى المشاركة البناءة التي تمثل الشكر الحقيقي، فأنا أعرف في قرارة نفسي أني لن أستطيع مجارات أستاذي الصادق خاصة في ظل ظروفي الحالية، لكني لن أدخر ان شاء الله أي جهد في المحاولة الا اذا تعدت المناقشة مستواي المتواضع وهذا ما أتوقعه من باقي الأعضاء أيضا.

استفسار:
سبق وأن بينت لنا أخي الصادق كيف أن التماثل يؤدي الى الثبات لكن سؤالي هو :
-هل يؤدي الثبات الى التماثل؟؟؟؟
فمثلا نقول أن قوانين الفيزياء اليوم هي نفسها البارحة ويعتقد علماء الفيزياء أنها نفسها عند بداية الكون، ومن جهة أخرى نقول أن قوانين الفيزياء على الأرض هي نفسها على القمر وهي نفسها في أي مكان من الكون
اذن نقول أن قوانين الفيزياء ثابتة في كل مكان وزمان (ملاحظة: أنا لم أعين حالة الراصد في ما مضى ومعنى كلامي السابق أن الراصد المتسارع على القمر مثلا يطبق نفس قوانين الراصد المتسارع على الشمس مثلا وهكذا دواليك...).
-هل يمكن أن نقول أن قوانين الفيزياء تماثلية؟؟؟؟

أخي الصادق أنا أعطيك الحق في تعديل أي مشاركة من مشاركاتي في حال شعرت أنها تخرجنا عن المسار الذي حددته للموضوع او تشكل أي تشويش على المفاهيم التى تريد الوصول اليها.


و السلام عليكم، في انتظار البقية.

الصادق
08-15-2009, 12:18 AM
حياك الله اخى Tyns19

بالفعل ان التفاعل والمشاركات البناءة تلعب دور كبير فى اثراء الموضوع كما انها تشجع الكاتب وتعطيه فكرة عن الخلفية العلمية للقارئ مما يساعد فى اختيار الامثلة المناسبة لتوصيل الفكرة

اخىTyns19 اذهب الله عنك التشتت وجعل ذهنك صافياً متقداً ببصيرة فيزيايئة فذة

لقد لمستُ فى مشاركاتك نضج الفكرة و سلاسة السياق الرياضى و العمق فى طرح الاسئلة.

سبق وأن بينت لنا أخي الصادق كيف أن التماثل يؤدي الى الثبات لكن سؤالي هو :
-هل يؤدي الثبات الى التماثل؟؟؟؟

نعم ان الثبات مؤشر يدل على وجود تماثل و الامثلة التى صيغتها تدل على ذلك و التماثل نتيجة للانقال الزمانى(قوانين الفيزياء اليوم هي نفسها البارحة ) يدل على انحفاظ الطاقة و التماثل نتيجة للانتقال المكانى (أن قوانين الفيزياء على الأرض هي نفسها على القمر وهي نفسها في أي مكان من الكون) يدل على انحفاظ كمية الحركة الخطية. والحالة العامة تعرف بنظرية نوزر و سوف نناقشها انشاء الله فى هذا الموضوع

هل يمكن أن نقول أن قوانين الفيزياء تماثلية؟؟؟؟

نعم هى كذلك و لكم مع وجود حالة واحدة حسب وجهة نظرى المتواضعة جداً لا تحقق هذا التماثل و هى قانون الثيرمودينمك الثانى لاننا لو عكسنا اتجاه الزمن فان الرصد سوف يلاحظ نقصان الانتروبى (شظايا كوب الشاى المتبعثرة على الارض سوف تتجمع و تستقيم كوباً من الشاى الحار على المنضدة ) و سوف يصرخ حينها ان هناك امر مريب وسوف يكتشف ان اتجاه الزمن قد عُكس (عندما ترجع شريط الفديو)

ولذلك سوف اجيب على سؤالك بحزر و اقول ان جميع القوانين الديناميكية تحترم (اى تحقق) قوانين التماثل


بالنسبة للفضاء الذى لا نعرف فيه نقطة اصل هو الفضاء الطورى فى الميكانيكا الاحصائية و ايضاً فضاء الهيئة فى الصياغة الهملتونية للميكانيكا التحليلية واى نقطة فى هذه الفضاءت تمثل حالة فيزيائية محددة للمنظومة

و الله اعلم

Tyns19
08-15-2009, 07:03 AM
السلام عليكم

اخىtyns19 اذهب الله عنك التشتت وجعل ذهنك صافياً متقداً ببصيرة فيزيايئة فذة
امين ان شاء الله، عنا وعنكم وعن كل المؤمنين.

لقد لمستُ فى مشاركاتك نضج الفكرة و سلاسة السياق الرياضى و العمق فى طرح الاسئلة.
ومنكم نستفيد أستاذي الكريم.
لقد غاب عن ذهني فضاء الهيئة وهذا لأني سرحت بأفكاري بعيدا، أما بالنسبة للجواب على السؤالين أعلاه فما زال عندي بعض الاستفسارات لكني لن أستبق الأحداث وسأناقش أفكاري في الوقت المناسب ان شاء الله وذلك بعد تقدم الموضوع بضع خطوات.
عندما وضعت التمرين السابق أدركت أن الموضوع سيرتبط بقوانين الانحفاظ ولو سطحيا، لكن من خلال اجابتك على سؤالي السابق أكاد أكون متأكد أن قوانين الانحفاظ هي محور الموضوع الرئيسي.(مجرد حدس).

الساعة الان هي الساعة الخامسة صباحا، والمشتقة الأولى لطاقتي الداخلية معدومة كما أن المشتقة الثانية تحمل اشارة موجبة ما يعني أنه وقت اعادة الشحن.................:rolleyes: :rolleyes: :rolleyes:
وفي الختام سلام، مع انتظار بقية الموضوع ...........

الصادق
08-18-2009, 08:00 AM
الصياغة الاجرانجية للميكانيكا الكلاسيكية

فضاء الهئية
افترض انه لدينا منظومة فيزيائية تتكون من N جسيم وهكذا لو اردنا ان نعرف مواقع هذه الجسيمات فى الفضاء x,y,x فاننا نحناج ان نعرف N متجهاً و مع مرور الزمن فان الجسيمات تغير مواقعها بالنسبة لنقطة ثابتة (نقطة الاصل) و بالتالى فان هذه المتجهات سوف تتغير مع الزمن. و لكن جميعاً ان التعامل مع المتجهات هو امر مرهق بعض الشئ خاصة فى حالتنا هذه اى حالة فضاء ثلالثى الابعاد به N متجه, لذا قام الفيزيائيون بايجاد فضاء قياسى يسمى فضاء الهئية و فى هذا الفضاء يحدد موقع اى جسيم بثلاثة احداثيات وبالتالى فان عدد الابعاد الكلى يساوى 3N و هذه الاحداثيات و التى يرمز لها ب http://latex.codecogs.com/gif.latex?\large%20q_1,%20q_2,%20.. .,q_{3N} نسمى بالاحداثيات المُعممة. ولكن ايضاً نعلم ان هذه الجسيمات تغير مواقعها مع مرور الزمن و لذلك طالما انه لدينا موقع الجسيم الان فاننا نحتاج فقط لسرعته حتى نحدد موقعه بعد مرور زمن t و هكذا نحتاج ان نُعرف لاى جسيم ثلاثة سرعات اى ان عدد السرعات الكلى للمنظومة هو 3N وهذه السرعات تسمى بالسرعات المُعممة ماهى الا تفاضل الاحداثيات المُعممة بالنسبة للزمن لذا نرمز لها ب http://latex.codecogs.com/gif.latex?\large%20\dot{q}_1,%20\do t{q}_2,%20...,\dot{q}_{3N} حيث النقطة اعلى الحرف تعنى التفاضل بالنسبة للزمن t. و هكذا فان المنظومة تحدد تماماً عن طريق الاحداثيات المُعممة والسرعات المُعممة فى فضاء الهئية

http://latex.codecogs.com/gif.latex?\huge%20(q_1,%20q_2,%20.. .,%20q_{3N};%20\dot{q}_1,%20\dot{q} _2,%20...,\dot{q}_{3N};t)

فضاء الطور
نعلم ان السرعة ترتبط بكمية الحركة و لذا فاننا احياناً نُعرف كميات حركة مُعممة بدلاً عن سرعات مُعممة (لاحظ ان السرعات المُعممة تعتمد على الاحداثيات المُعممة فلو اعطيتنى الاحداثيات المُعممة للمنظومة فاننى سوف اقاضلها بالنسبة للزمن و اعطيك السرعات المُعممة) وكميات الحركة المُعممة هذه يرمز لها بالرمز http://latex.codecogs.com/gif.latex?\large%20p_1,%20p_2,%20.. .,%20p_{3N} ويمكن اعتبارها مستقلة عن الاحداثيات المُعممة و بالتالى فان المنظومة تحدد عن طريق 6N بعد فى فضاء يسمى بفضاء الطور

http://latex.codecogs.com/gif.latex?\huge%20(q_1,%20q_2,%20.. .,%20q_{3N}, p_1,%20p_2,%20...,%20p_{3N}%20;t)

واى نقطة فى هذا الفضاء http://latex.codecogs.com/gif.latex?\large%20(q_i,%20p_i) تمثل موقع و كمية تحرك الجسيم و بالتالى فان النقاطة فى الفضاء الطورى هى عبارة حالة الجسيم عند لحظة زمنية معينة.


يتبع.......

الصادق
08-18-2009, 09:23 AM
دالة لاجرانج:

دالة لاجرانج هى دالة تعتمد على الاحداثيات المُعممة والسرعات المُعممة وهى على انها الفرق بين طاقة الحركة و طاقة الجهد

http://latex.codecogs.com/gif.latex?\huge%20L(q_i,%20\dot{q}_ i)=T(\dot{q}_i)-V(q_i)%20\qquad%20(6)

مبدأ الفعل الاقل

عندما تتطور المنظومة مع مرور الزمن فان حركتها تشكل مساراً فى فضاء الهيئة وذلك لان الجسيم يكون له موقع وسرعة محددة فى فضاء الهيئة ومع مرور الزمن يتغير موقعه و سرعته وبالتالى ينتقل من نفطة الى اخرى فى فضاء الهيئة. الان افترض ان المنظومة عند لحظة معينة t_i كانت عند النقطة http://latex.codecogs.com/gif.latex?\huge%20(q_i,%20\dot{q}_i ) فى فضاء الهيئة ولاحقاً عند لحظة زمنية t_f وصلت المنظومة الى نقطة اخرى فى فضاء الهيئة و هى http://latex.codecogs.com/gif.latex?\huge%20(q_f,%20\dot{q}_f ). الان هناك عدد لانهائى من المسارات التى تربط بين هاتين النقطتين و السؤال هو ماهو المسار الذى انتقلت خلاله المنظومة؟ للاجابة على هذا السؤال دعنا نُعرف عدداً S يسمى بالفعل و هذا الفعل يُعرف على انه التكامل الزمنى لدالة لاجرانج من لحظة بداية الحركة t_i الى لحظة نهاية الحركة t_f اى انه يُعرف بالتكامل التالى:

http://latex.codecogs.com/gif.latex?\huge%20S[q]=\int_{t_i}^{t_f}dt%20L(q_i,\dot{q} _i,t)\qquad%20(7)

الان ينص مبدأ الفعل الاقل على ان المسار الحقيقى الذى تتحرك خلاله المنظومة هو المسار الذى يجعل للفعل S اقل قيمة ممكنة.

معادلات اويلر- لاجرانج:

نعلم انه عند النهاية الصغرى ان المشتقة الاولى تساوى صفراً و هكذا طالما ان لدالة الفعل نهاية صغرى عند المسار الحقيقى للمنظومة فى فضاء الهيئة فان التغير فيها يجب ان يساوى الصفر اى ان

http://latex.codecogs.com/gif.latex?\huge%20\delta%20S[q]=\int_{t_i}^{t_f}dt%20\left(\frac{\ partial%20L}{\partial%20q_i}-\frac{d}{dt}\frac{\partial%20L}{\pa rtial%20\dot{q}_i}%20\right%20)\del ta%20q_i=0

وهكذا طالما ان التغير فى الاحداتى المُعمم q هو تغير اعتباطى فان الحد بين القوسين فى المعادلة الاخيرة يجب ان يساوى صفراً, اى ان

http://latex.codecogs.com/gif.latex?\huge%20\frac{\partial%20 L}{\partial%20q_i}-\frac{d}{dt}\left(%20\frac{\partial %20L}{\partial%20\dot{q}_i}\right%2 0)%20=0%20\qquad%20(8)

وهذه المعادلة تمثل معادلة حركة المنظومة و تسمى بمعادلات اويلر لاجرانج

مثال:

اوجد معادلة الحركة لجسيم كتلته m يتحرك فى حقل جهد http://latex.codecogs.com/gif.latex?\huge%20V(q)؟

الحل :

اولاً نوجد دالة لاجرانج
ثانياُ نطبق معادلة اويلر لاجرانج

دالة لاجرانج هى عبارة عن الفرق بين طاقة الحركة و طاقة الجهد

http://latex.codecogs.com/gif.latex?\huge%20L=\frac{1}{2}m\do t{q}^2-V(q)\qquad%20(9)

حيث عوضنا طاقة الحركة تساوى نصف الكتلة مضروبة فى مربع السرعة. الان نوجد تفاضل دالة لاجرانج بالنسبة للسرعة المُعممة

http://latex.codecogs.com/gif.latex?\huge%20\frac{\partial%20 L}{\partial%20\dot{q}}=m\dot{q}

وهكذا نرى ان تفاضل دالة لاجرانج بالسبة للسرعة المعممة يعطى كمية الحركة المُعممة (الكتلة مضروبة فى السرعة)

دعنا نحسب تفاضل النتيجة الاخيرة بالنسبة للزمن

http://latex.codecogs.com/gif.latex?\huge%20\frac{d}{dt}\left (%20\frac{\partial%20L}{\partial%20 \dot{q}}\right%20)=m\ddot{q}

حيث تفاضل السرعة بالنسبة للزمن هو عبارة عن التسارع و قد رمزنا له بq عليها نقطتان اى انه عبارة عن التفاضل الثانى للحداثى q
اخيراً نحسب تفاضل دالة لاجرانج بالنسبة للاحداثى q وطالما ان طاقة الحركة لاتعتمد على q فان التفاضل سوف يؤثر على دالة الجهد فقط اى ان

http://latex.codecogs.com/gif.latex?\huge%20\frac{\partial%20 L}{\partial%20q}=-\frac{\partial%20V}{\partial%20q}=F

حيث اننا عوضنا تفاضل دالة الجهد بالنسبة للاحداثى تمثل القوة المؤثرة على الجسيم نتيجة لوجوده فى ذلك الجهد

الان بالتعويض فى معادلة اويلر لاجرانج (8) نجد ان

http://latex.codecogs.com/gif.latex?\huge%20\\\frac{\partial% 20L}{\partial%20q}-\frac{d}{dt}\left(%20\frac{\partial %20L}{\partial%20\dot{q}}\right%20) %20=0%20\\%20\\%20F-m\ddot{q}=0\\%20\\%20\rightarrow%20 F=m\ddot{q}

اى ان القوة تساوى الكتلة فى التسارع و هذه المعادلة هى عبارة عن قانون نيوتن الثانى للحركة


يتبع.......

الصادق
08-18-2009, 10:57 AM
الصياغة الهملتونية للميكانيكا الكلاسيكية
قلنا ان دالة لاجرانج هى دالة فى الاحداثيات المُعممة و السرعات المُعممة و الزمن. والان نريد ان نوجد تحويل لجندر لدالة لاجرانج لنحصل على دالة جديدة تعتمد على الاحداثيات المُعممة و كميات الحركة المُعممة و الزمن و هذه الدالة تسمى بدالة هملتون و تعطى ب

http://latex.codecogs.com/gif.latex?\huge%20H(q_i,p_i,t)=p_i\ dot{q}_i-L(q_i,\dot{q}_i,t)\qquad%20(9)

معادلة هملتون القانونية للحركة

دعنا نوجد التغير التام فى دالة هملتون. ومن اجل هذا الغرض سوف نستخدم قاعدة السلسلة فى علم الحسبان اى اننا سوف نكنب التفاضل التام على انه مجموع التفاضلات الجزيئة و هكذا طالما ان H تعتمد على q_i و p_i و t فان التغير التام يعطى ب

http://latex.codecogs.com/gif.latex?\huge%20dH=\frac{\partial %20H}{\partial%20q_i}dq_i+\frac{\pa rtial%20H}{\partial%20p_i}dp_i%20+% 20\frac{\partial%20H}{\partial%20t} dt\qquad%20(10)

اما من الجانب الاخر اذا اوجدنا التغير التام ل H من المعادلة (9) فاننا سوف نحصل على

http://latex.codecogs.com/gif.latex?\huge%20\\dH=d(p_i\dot{q} _i)-dL\\%20\\%20dH=p_i%20d\dot{q}_i+\do t{q}_idp_i-\left(%20\frac{\partial%20L}{\parti al%20q_i}dq_i+\frac{\partial%20L}{\ partial%20\dot{q}_i}d\dot{q}_i%20+% 20\frac{\partial%20L}{\partial%20t} dt\right%20)
ومن معادلة اويلر - لاجرانج يمكننا كتابة الحد الاول داخل القوس فى المعادلة الاخيرة على النحو التالى

http://latex.codecogs.com/gif.latex?\huge%20\frac{\partial%20 L}{\partial%20q_i}=\frac{d}{dt}\lef t(\frac{\partial%20L}{\partial%20\d ot{q}_i}%20\right%20)

فتصبح المعادلة
http://latex.codecogs.com/gif.latex?\huge%20dH=p_i%20d\dot{q} _i+\dot{q}_idp_i-\left(%20\frac{d}{dt}\left(\frac{\p artial%20L}{\partial%20\dot{q}_i}\r ight)%20dq_i+\frac{\partial%20L}{\p artial%20\dot{q}_i}d\dot{q}_i%20+%2 0\frac{\partial%20L}{\partial%20t}d t\right%20)


ولكن فى المثال السابق قد راينا ان تفاضل دالة لاجرانج بالنسبة للسرعة المُعممة يمثل كمية الحركة المُعممة اى ان

http://latex.codecogs.com/gif.latex?\huge%20\\%20dH=p_i%20d\d ot{q}_i+\dot{q}_idp_i-\left(%20\dot{p}_i%20dq_i+p_id\dot{ q}_i%20+%20\frac{\partial%20L}{\par tial%20t}dt\right%20)\\%20\\%20dH=\ dot{q}_i%20dp-\dot{p}_idq_i-\frac{\partial%20L}{\partial%20t}dt %20\qquad(11)

حيث رمزنا لتفاضل كمية الحركة المُعممة بالنسبة للزمن ب p منقوطة. الان دعنا نقار المعادلة (11) بالمعادلة (10) ونلاحظ انه طالما ان q و p و t مستقلة عن بعضها البعض فان فان المعاملات المضروبة فى dq_i وفى dp_i و فى dt يجب ان تتساوى اى ان

http://latex.codecogs.com/gif.latex?\huge%20\\%20\dot{q}_i=\f rac{\partial%20H}{\partial%20p_i}%2 0\qquad%20\qquad%20\dot{p}_i=-\frac{\partial%20H}{\partial%20q_i} %20\qquad%20\qquad%20\frac{\partial %20L}{\partial%20t}=-\frac{\partial%20H}{\partial%20t}\q quad(12)

وهذه هى معادلة هملتون القانونية للحركة وهى معادلة من الدرجة الاولى على خلافل معادلة اويلر لاجرانج و التى هى معادلة من الدرجة الثانية

مثال:
اوجد معادلة الحركة لجسيم يتحرك فى مجال دالة جهد تعتمد فقط على الاحداثى المُعمم q مستخدماً الصياغة الهملتونية.

الحل:
فى المثال السابق اوجدنا دالة لاجرانج و كانت تساوى
http://latex.codecogs.com/gif.latex?\huge%20L=\frac{1}{2}m\do t{q}^2-V(q)\qquad%20(9)

الان دالة هملتون تُعطى ب

http://latex.codecogs.com/gif.latex?\huge%20\\H=p\dot{q}-L%20\\\\%20H=p\dot{q}-\frac{m\dot{q}^2}{2}+V(q)

ولكن طالما ان دالة هملتون هى دالة فى كمية الحركة وليس فى السرعة فاننا سوف نكتب السرعة بدلالة كمية الحركة لنحصل على

http://latex.codecogs.com/gif.latex?\huge%20H=\frac{p^2}{2m}+ V(q)\qquad%20(13)

الان بتطبيق معادلة هملتون القانونية نحصل على

http://latex.codecogs.com/gif.latex?\huge%20\\%20\dot{q}=\fra c{\partial%20H}{\partial%20p}=\frac {p}{m}%20\\%20\\%20-\dot{p}=\frac{\partial%20H}{\partia l%20q}=\frac{\partial%20V}{\partial %20q}%20\\%20\\%20\rightarrow%20F=\ dot{p}

اى ان القوة تساوى معدل تغير كمية الحركة بالنسبة للزمن و هذا هو قانون نيوتن الثانى للحركة

وهكذا نلاحظ من هذا المثال و المثال السابق ان الصياغة اللاجرانجية و الصياغة الهملتونية تؤول الى الصياغة النيوتونية و لكن نجد ان الصياغة اللاجرانجية والهملتونية اكثر كفاءة من الصياغة النيوتونية فى التعامل مع الانظمة المعقدة


يتبع......ز

Tyns19
08-19-2009, 01:13 AM
شكرا أخي الصادق على المجهود، جعله الله في ميزان حسناتك
لقد زدتنا تشويقا لمتابعة باقي الموضوع

المتأمل
08-19-2009, 01:34 PM
بارك الله أخي الكريم و زادك علما و حكمة

تغريد
08-21-2009, 08:40 PM
بارك الله فيك و في جهودك أخي الكريم الصادق
و يسر الله لك أمرك و وفقك فيما شغلت به

و نحن بانتظار إكتمال هذا الموضوع و الذي سبدو مشوقا و مفيدا

و كل عام و أنتم و جميع أعضاء المنتدى بألف خير بمناسبة حلول شهر رمضان المبارك

جعلنا الله واياكم و جميع أعضاء هذا المنتدى الكريم من صوامه وقوامه

وجعلنا ان شاء الله من العتقاء من النار

الصادق
08-23-2009, 07:46 PM
التماثل الزمنى (التماثل تحت تحويل الانتقال الزمنى) وانحفاظ الطاقة الكلية

افترض انه كان لدينا تماثل تحت تحويل الانتقال الزمنى (بمعنى ان المنظومة متماثلة عند جميع اللحظات الزمنية) وهكذا فان المنظومة لن تعتمد على الزمن بشكل صريح اى ان دالة الهملتونيان بدورها لن تعتمد على الزمن صراحةً

البرهان:

اذا كانت http://latex.codecogs.com/gif.latex?\huge%20H(t)=H(t+\tau)
حيث تاو انتقال زمنى اعتباطى (اختيارى)

بايجاد مفكوك تايلور حول اللحظة الزمنية t نحصل على
http://latex.codecogs.com/gif.latex?\huge%20H(t)=H(t)+\tau\fr ac{\partial%20H}{\partial%20t}+\fra c{\tau^2}{2!}\frac{\partial^2%20H}{ \partial%20t^2}+\dots

وكما قلنا سابقاً فان هذا يقود الى ان التفاضل الزمنى الجزئى لدالة هملتون يساوى صفراً
http://latex.codecogs.com/gif.latex?\huge%20\frac{\partial%20 H}{\partial%20t}=0


الان دعنا نحسب التفاضل الزمنى الكلى (التام) لدالة هملتون مستخدمين قاعدة السلسلة

http://latex.codecogs.com/gif.latex?\huge%20\\%20\frac{dH}{dt }=\frac{\partial%20H}{\partial%20q_ i}\frac{dq_i}{dt}+\frac{\partial%20 H}{\partial%20p_i}\frac{dp_i}{dt}+\ frac{\partial%20H}{\partial%20t}\\\ \%20\\%20\frac{dH}{dt}=\frac{\parti al%20H}{\partial%20q_i}\dot{q_i}+\f rac{\partial%20H}{\partial%20p_i}\d ot{p_i}+\frac{\partial%20H}{\partia l%20t}%20\qquad%20(14))

ولكن من معادلات هملتون القانونية نعلم ان تفاضل دالة هملتون بالنسبة لكمية الحركة يمثل السرعة بينما ان تفاضل دالة هملتون بالنسبة للاحداثى المكانى يمثل سالب القوة اى تفاضل التسارع بالنسبة للزمن (انظر المعادلات (12)

وبالتعويض فى المعادلة (14) مع ملاحظة ان التفاضل الجزئى لدالة هملتون يساوى صفر, سوف نحصل على

http://latex.codecogs.com/gif.latex?\huge%20\frac{dH}{dt}=-\dot{p}_i\dot{q_i}+\dot{q}_i\dot{p_ i}=0


وهذا يعنى ان الطاقة الكلية (الهملتونيان) تظل ثابتة مع مرور الزمن
http://latex.codecogs.com/gif.latex?\huge%20H=constant

اذن من هذا نخلص الى: اذا كانت المنظومة متماثلة تحت تحويل الانتقال الزمنى فان الطاقة تظل ثابتة (محفوظة). وهكذا نلاحظ ان قانون انحفاظ الطاقة ماهو الا انعكاس للتماثل الزمنى


التماثل المكانى (التماثل تحت الانتقال المكانى) و انحفاظ كمية الحركة الخطية

لو كانت المنظومة متماثلة تحت تحويل الانتقال المكانى فان هذا يعنى ان دالة هملتون لا تعتمد على الاحداثى المكانى اعتماداً صريحاً (برهن!!!) الان من معادلات هملتون القانونية نجد ان

http://latex.codecogs.com/gif.latex?\huge%20\dot{p}_i=-\frac{\partial%20H}{\partial%20q_i} =0\Rightarrow%20\frac{dp_i}{dt}=0

وهكذا فان كمية الحركة تظل ثابتة مع مرور الزمن

http://latex.codecogs.com/gif.latex?\huge%20p_i=constant

ولذلك نقول انه لو كانت المنظومة متماثلة مكانياً فان كمية الحركة الخطية تظل ثابتة (محفوظة) مما يعنى ان قانون انحفاظ كمية الحركة هو انعكاس للتماثل المكانى


يتبع......

الصادق
08-24-2009, 01:52 AM
اخى Tyns19 شكر الله لك وبارك فيك وضاعف لك الثواب والاجر

اخى المتأمل وبارك الله فيك وذادك حكمةً وعلماً اضعاف مضاعفة

اختى الكريمة تغريد بارك الله فيك و يسر امرك وتقبل الله منا ومنكم صالح الاعمال

ورمضان كريم و مبارك على الجميع

الصادق
08-28-2009, 04:44 AM
اقواس بوايسون

تعتبر اقواس بوايسون عبارة عن وصف جبرى للميكانيكا الكلاسيكية (هى نفسها تكاد تنطبق على اقواس التبادلية فى ميكانيكا الكم)

التعريف:

افترض ان http://latex.codecogs.com/gif.latex?\large%20A(q,p)\;%20,\;%2 0B(q,p) هما عبارة عن دوال فى فضاء الطور (فى ميكانيكا الكم نعتبرهما مؤاثرات فى فضاء هيلبيرت ونستبدل اقواس بوايسون باقواس التبادلية) الان نُعرف قوسا بوايسون http://latex.codecogs.com/gif.latex?\large%20\{A,B\}_p بالعلاقة التالية

http://latex.codecogs.com/gif.latex?\huge%20\{A,B\}_p=\sum_{k =1}^{N}\left[%20\frac{\partial%20A}{\partial%20q _k}\;\frac{\partial%20B}{\partial%2 0p_k}-\frac{\partial%20B}{\partial%20q_k} \;\frac{\partial%20A}{\partial%20p_ k}\right%20]\qquad%20(15)



خواص اقواس بوايسون:

1- خاصية ضد التماثلية

http://latex.codecogs.com/gif.latex?\huge%20\{A,B\}_p=-\{B,A\}_P%20\qquad%20(16)
2-الخاصية الخطية

http://latex.codecogs.com/gif.latex?\huge%20\{\alpha%20A+%20\ beta%20B,C\}_p=\alpha%20\{A,C\}_p+\ beta%20\{B,C\}_p%20\qquad%20(17)

3-خاصية لايبنز
http://latex.codecogs.com/gif.latex?\huge%20\{AB,C\}_p=A\{B,C \}_p+\{A,C\}_p%20B%20\qquad%20(18)

4-متطابقة جاكوبى

http://latex.codecogs.com/gif.latex?\huge%20\{A,\{B,C\}_p\}_p +\{B,\{C,A\}_p\}_p+\{C,\{A,B\}_p\}_ p=0%20\qquad%20(19)

تمرين: مستخدماً التعريف (15) برهن خواص اقواس بوايسون (16) و (17) و (18) و (19)

الان تلاحظ ان خواص اقواس بوايسون هى نفسها خواص جبر المصفوفات بالنسبة لتبادلية المصفوفات http://latex.codecogs.com/gif.latex?\large%20[A,B]=AB-BA وايضاً هى نفسها خواص التفاضل d (ولهذا السبب نجد ان فى ميكانيكا الكم هناك وصفين مستقلين وهما الوصف المصفوفى وهو وصف هايزنبيرج (ميكانيكا المصفوفات) و الوصف التفاضلى (الموجى) وهو وصف شرودنجر (ميكانيكا الموجات)


مثال: احسب قوسا بوايسون للموقع وكمية الحركة http://latex.codecogs.com/gif.latex?\huge%20\{q_i,p_j\}_p

الحل: بتعويض http://latex.codecogs.com/gif.latex?\large%20A=q_i و http://latex.codecogs.com/gif.latex?\large%20B=p_j فى تعريف قوسا بوايسون فى المعادلة (15) نحصل على

http://latex.codecogs.com/gif.latex?\huge%20\{q_i,p_j\}_p=\su m_{k=1}^{N}\left(\frac{\partial%20q _i}{\partial%20q_k}\;\frac{\partial %20p_j}{\partial%20p_k}-\frac{\partial%20p_j}{\partial%20q_ k}\;\frac{\partial%20q_i}{\partial% 20p_k%20}%20\right%20)

طالما ان الاحداثيات المكانية مستقلة خطياً عن كميات الحركة فان تفاضل q بالنسبة ل p يساوى صفر و ايضاً تفاضل p بالنسبة لq يساوى الصفر, وهكذا فان الحد الثانى فى الطرف الايمن من المعادلة الاخيرة يساوى صفراً

http://latex.codecogs.com/gif.latex?\huge%20\{q_i,p_j\}_p=\su m_{k=1}^{N}\left(\frac{\partial%20q _i}{\partial%20q_k}\;\frac{\partial %20p_j}{\partial%20p_k}%20\right%20 )

الان تفاضل http://latex.codecogs.com/gif.latex?\large%20q_i بالنسبة ل http://latex.codecogs.com/gif.latex?\large%20q_k يساوى و احد فى حالة كانت i=k وعندها نكون قد فاضلنا دالة بالنسبة لنفسها اما اذا كانت i لا تساوى k فان التفاضل سوف يساوى صفراً لاننا حينها نكون فاضلنا بعد احداثى بالنسبة لاحداثى اخر مستقل عنه. وهكذا فان

http://latex.codecogs.com/gif.latex?\huge%20\frac{\partial%20 q_i}{\partial%20q_k}=\left\{\begin{ matrix}%201%20&%20i=k\\%200%20&%20i\neq%20k%20\end{matrix}\right.

هناك دالة تُعرف بدلتا كرونكر و يرمز لها بالرمز بالحرف دلتا http://latex.codecogs.com/gif.latex?\huge%20\delta_{ik} وهى تساوى وحد فى حال كانت i=k وتساوى صفراً عند i لا تساوى k اى ان

http://latex.codecogs.com/gif.latex?\huge%20\delta_{ik}=\left \{\begin{matrix}%201%20&%20i=k\\%200%20&%20i\neq%20k%20\end{matrix}\right.

وهكذا فان التفاضل اعلاه يساوى دالة كرونكر

http://latex.codecogs.com/gif.latex?\huge%20\frac{\partial%20 q_i}{\partial%20q_k}=\delta_{ik}


وبنفس المنطلق نجد ان تفاضل http://latex.codecogs.com/gif.latex?\large%20p_j بالنسبة ل http://latex.codecogs.com/gif.latex?\large%20p_k يساوى الواحد فى حال تساوى المعاملات ويساوى صفراً فى ما عدا ذلك و هكذا فاننا سوف نحصل على دالتا كرونكر مرة اخرى

http://latex.codecogs.com/gif.latex?\huge%20\frac{\partial%20 p_j}{\partial%20p_k}=\delta_{ik}

الان بالتعويض فى قوسا بوايسون اعلاه نحصل على

http://latex.codecogs.com/gif.latex?\huge%20\{q_i,p_j\}_p=\su m_{k=1}^{N}\left(\delta_{ik}\delta_ {jk}%20\right%20)

طالما اننا سوف نجمع جميع القيم الممكنة ل k فان دلتا كرونكر الثانية سوف تساوى 1 فقط عند قيمة k التى تساوى j وهكذا سوف نعوض k ب j فى جميع اجزاء العلاقة الاخيرة مما يجعل دلتا كرونكر الثانية مساوية لواحد و يصيح لدينا دالتا بين i و j اى ان

http://latex.codecogs.com/gif.latex?\huge%20\{q_i,p_j\}_p=\de lta_{ij}\qquad(20)

ونلاحظ ان هذه العلاقة تشبة اقواس التبادلية بين مؤثر الموقع و كمية الحركة فى ميكانيكا الكم

يتبع............

Tyns19
08-28-2009, 08:33 AM
السلام عليكم
سأحاول برهنة الخواص 16،17،18،19 وذلك باستخدام التعريف 15:
ملاحظة: من أجل التبسيط سأبرهن الخواص السابقة من أجل درجة حرية وحيدة وهذا لا ينقص من البرهان أي شيء بل سيبقى البرهان شامل للحالة العامة التي تبرهن بنفس الطريقة لهذا فأنا أستعمل التعريف 15 في الصورة التالية:

http://www.codecogs.com/eq.latex?\150dpi%20 \{A,B\}_p=\frac{\partial A}{\partial q}\;\frac{\partial B}{\partial p}-\frac{\partial B}{\partial q}\;\frac{\partial A}{\partial%20p}
استعملت درجة حرية وحيدة حتى لا نكتب في كل مرة عبارة المجموع و الأندكس k وما الى غير ذلك، وطريقة البرهان في الحالة العامة هي نفسها.

1- خاصية ضد التماثلية:
حسب التعريف يمكن أن نكتب:

http://www.codecogs.com/eq.latex?\150dpi%20 \{B,A\}_p=\frac{\partial B}{\partial q}\;\frac{\partial A}{\partial p}-\frac{\partial A}{\partial q}\;\frac{\partial B}{\partial p}=-\left( \frac{\partial A}{\partial q}\;\frac{\partial B}{\partial p}-\frac{\partial B}{\partial q}\;\frac{\partial A}{\partial p} \right)=-\{A,B\}_p
وبهذا نكون قد برهنا الخاصية الأولى#.

2-الخاصية الخطية:

http://www.codecogs.com/eq.latex?\150dpi%20 \{\alpha A+ \beta B,C\}_p=\frac{\partial R}{\partial q}\frac{\partial C}{\partial p}-\frac{\partial R}{\partial p}\frac{\partial C}{\partial q}
حيث:

http://www.codecogs.com/eq.latex?\150dpi%20\ R=\alpha A+ \beta B\Rightarrow \left\{\begin{matrix}\frac{\partial R}{\partial q}=\alpha \frac{\partial A}{\partial q}+\beta \frac{\partial B}{\partial q}\\\frac{\partial R}{\partial p}=\alpha \frac{\partial A}{\partial p}+\beta \frac{\partial B}{\partial p}\end{matrix}\right.
بالتعويض واعادة الترتيب نجد:

http://www.codecogs.com/eq.latex?\120dpi%20 \{\alpha A+ \beta B,C\}=\alpha\left(\frac{\partial A}{\partial q}\frac{\partial C}{\partial p}-\frac{\partial A}{\partial p}\frac{\partial C}{\partial q}\right)+\beta\left(\frac{\partial B}{\partial q}\frac{\partial C}{\partial p}-\frac{\partial B}{\partial p}\frac{\partial C}{\partial q}\right)\\
اذن:

http://www.codecogs.com/eq.latex?\150dpi%20 \{\alpha A+ \beta B,C\}_p=\alpha \{A,C\}_p+\beta \{B,C\}_p
وهو برهان الخاصية الثانية#.

3-خاصية لايبنز :

http://www.codecogs.com/eq.latex?\150dpi%20 \{AB,C\}_p= \frac{\partial }{\partial q}(AB)\;\frac{\partial C}{\partial p}-\frac{\partial }{\partial p}(AB)\;\frac{\partial C}{\partial q}
نعلم أن:

http://www.codecogs.com/eq.latex?\150dpi%20 \frac{\partial }{\partial x}(fg)=g\frac{\partial f}{\partial x}+f\frac{\partial g}{\partial x}
اذن:

http://www.codecogs.com/eq.latex?\120dpi%20 \{AB,C\}_p=\left(A\frac{\partial B}{\partial q}+B\frac{\partial A}{\partial q}\right)\;\frac{\partial C}{\partial p}-\left(A\frac{\partial B}{\partial p}+B\frac{\partial A}{\partial p}\right)\;\frac{\partial C}{\partial q}
وبعد اعادة الترتيب نجد:

http://www.codecogs.com/eq.latex?\120dpi%20 \{AB,C\}_p=A\left(\frac{\partial B}{\partial q}\;\frac{\partial C}{\partial p}-\frac{\partial B}{\partial p}\;\frac{\partial C}{\partial q}\right)+\left(\frac{\partial A}{\partial q}\;\frac{\partial C}{\partial p}-\frac{\partial A}{\partial p}\;\frac{\partial C}{\partial q}\right)B
وهذا معناه أن:

http://www.codecogs.com/eq.latex?\120dpi%20 \{AB,C\}_p=A\{B,C\}_p+\{A,C\}_p B
وهكذا نكون قد برهنا الخاصية الثالثة#.

4-متطابقة جاكوبى:
بتطبيق التعريف 15 على أي ثلاثية كالتالي http://latex.codecogs.com/gif.latex?\{L,\{M,N\}_p\}_p نجد العلاقة التالية التي سأسميها مؤقتا العلاقة الثلاثية:

http://www.codecogs.com/eq.latex?\120dpi%20 \{L,\{M,N\}_p\}_p=\frac{\partial L}{\partial q}\;\left(\frac{\partial^2M }{\partial p\;\partial q}\;\frac{\partial N}{\partial p}+ \frac{\partial M}{\partial q}\; \frac{\partial^2N}{\partial p^2} -\frac{\partial^2M }{\partial p^2}\;\frac{\partial N}{\partial q}- \frac{\partial M}{\partial p}\; \frac{\partial^2N}{\partial q \;\partial p} \right)
http://www.codecogs.com/eq.latex?\120dpi%20\ -\frac{\partial L}{\partial p}\;\left(\frac{\partial^2M }{\partial q^2}\;\frac{\partial N}{\partial p}+ \frac{\partial M}{\partial q}\; \frac{\partial^2N}{\partial q\;\partial p} -\frac{\partial^2M }{\partial q\partial p}\;\frac{\partial N}{\partial q}- \frac{\partial M}{\partial p}\; \frac{\partial^2N}{\partial q^2} \right)\\
يمكن الان الحصول على أي ثلاثية من متطابقة جاكوبي، مثلا كي نحصل على الثلاثية http://latex.codecogs.com/gif.latex?\{B,\{C,A\}_p\}_p نضع في المعادلة السابقة L=B، M=C، N=A إذن يمكننا الان ايجاد الثلاثيات الثلاث الموجودة في متطابقة جاكوبي، وبعد التعويض سنجد أن متطابقة جاكوبي محققة.(لن أعوض أنا هنا لكبر العلاقة التي ستنتج، لكني أنصح الجميع بالتجربة على الورق).
وبهذا نكون قد برهنا على صحة متطابقة جاكوبي#.

ملاحظة: أرجو من الجميع التأكد من أن العلاقة الثلاثية صحيحة (أنا أخشى الأخطاء الكتابية لقد عملت كل جهدي لنقلها كما هي من الورق لكن مع كل هذه المشتقات من يدري) لأني قد أكون نسيت حرف أو حرفين أثناء الطباعة، المهم طريقة البرهان صحيحة وأنا قد قمت بها على الورق بدقة تامة والنتيجة مضمونة.

ملاحظة 2: في البرهان على الخاصية الخطية alpha و beta ثابتان لا يتعلقان بالمتغيرات الديناميكية وهو ما أنتظر تأكيده من أستاذي الصادق (وان كنت على أتم الثقة على أنه هذا هو الحال).

اذا كان لدى أحدكم برهان على الخاصة الأخيرة غير هذا سأكون سعيد جدا برؤيته.

والله أعلم.
و الشكر لك كل الشكر يا أستاذنا الصادق.

في انتظار باقي الموضوع ان شاء الله..........

Tyns19
08-28-2009, 09:35 AM
سؤال بسيط:
برهن أن:

http://www.codecogs.com/eq.latex?\120dpi%20 \begin{matrix}\{q_i,q_j\}_p=0\\\{p_ i,p_j\}_p=0\end{Bmatrix}\qquad\fora ll\; (i,j)
السؤال ليس موجه لك أخي الصادق، بل هو موجه لبقية الأعضاء.

الصادق
08-28-2009, 07:51 PM
هذا عظيم جداً

حلول صحيحة و منسقة ومتكاملة شكراً لك اخى Tyns19 على هذا المجهود والمتابعة
واضم صوتى لصوتك واتمنى ان يجد سؤالك فى المشاركة السابقة من يجيب عليه من اعضاء المنتدى الكرام

بالنسبة لالفا وبيتا فهما ثوابت حقيقية لا تعتمد على المتغيرات الديناميكية و حدثك سليم اخى وانا اؤكد عليه


شكر الله لك وجزاك خيراً

تغريد
08-28-2009, 10:22 PM
جهد تشكر عليه أخي Tyns19 وفقك الله

من الواضح أن كلاهما صفر لأننا في الحالة الأولى سنفاضل q بالنسبة ل p في كل حد من الحدود و هذا طبعا يعطي صفرا أيا كانت محاور الاحداثيات و العكس بالعكس

و اسمحوا لي بأن أتساءل
حول قول أخي الكريم الصادق
(طالما ان الاحداثيات المكانية مستقلة خطياً عن كميات الحركة )
أعتقد أن هذا دوما يعتبر هذا صحيحا في الميكانيكا الكلاسيكية
و أعتقد أن هذا غير صحيح في ميكانيكا الكم
و لكن هل يدل هذا على عدم استقلال الاحداثيات المكانية عن كميات الحركة
نعلم أن مبدأ عدم اليقين متعلق بمتجه الموضع و العزم و لكن
لكن الحديث هنا يتعلق بمحاور الإحداثيات لأن هذا يدل على الإحداثيات هي غير مستقلة
ما دلالات ذلك؟؟؟؟؟؟؟
يبدو ان الأمور اختلطت على
؟؟؟؟؟

الصادق
08-29-2009, 06:50 AM
جهد تشكر عليه أخي Tyns19 وفقك الله

من الواضح أن كلاهما صفر لأننا في الحالة الأولى سنفاضل q بالنسبة ل p في كل حد من الحدود و هذا طبعا يعطي صفرا أيا كانت محاور الاحداثيات و العكس بالعكس

و اسمحوا لي بأن أتساءل
حول قول أخي الكريم الصادق
(طالما ان الاحداثيات المكانية مستقلة خطياً عن كميات الحركة )
أعتقد أن هذا دوما يعتبر هذا صحيحا في الميكانيكا الكلاسيكية
و أعتقد أن هذا غير صحيح في ميكانيكا الكم
و لكن هل يدل هذا على عدم استقلال الاحداثيات المكانية عن كميات الحركة
نعلم أن مبدأ عدم اليقين متعلق بمتجه الموضع و العزم و لكن
لكن الحديث هنا يتعلق بمحاور الإحداثيات لأن هذا يدل على الإحداثيات هي غير مستقلة
ما دلالات ذلك؟؟؟؟؟؟؟
يبدو ان الأمور اختلطت على
؟؟؟؟؟


نعم هذا دائماً صحيح فى فضاء الطور فى الميكانيكا الكلاسيكية و لكن فى ميكانيكا الكم لا تعتبر كمية الحركة كاحد الابعاد لاننا فى ميكانيكا الكم نتحدث عن فضاء اقليدى به ثلاثة ابعاد x و y و z اما مؤثر كمية الحركة ومؤثر الموقع (يتطابق مع الموقع فى حال التمثيل المكانى ) فهما عبارة عن مؤثرات تؤثر على متجهات الحالة فى فضاء هيلبيرت

اذن اما ان نعتبر الاحداثيات المكانية عبارة عن ابعاد وهكذا فان كمية الحركة (ليست بعداً) عبارة عن مؤثر يتناسب مع المؤثر التفاضلى المكانى او ان نعتبر كميات الحركة عبارة عن ابعاد و ان الموقع (ليس بعداً ) عبارة عن مؤثر يتناسب مع المؤثر تفاضلى بالنسبة لكمية الحركة

مبدأ هايزنبيرج فى ميكانيكا الكم ينجم عن التبادلية بين المؤثرات المترافقة ( المؤثرات ليست دوال وانما هى تؤثر على الدوال) ولكن تعريف التبادلية لا يستوجب ان تكون المؤثرات دوال فى الفضاء الطورى وتعريف التبادلية هو AB-BA



الفرق بين احداثيات الموقع كاسس للفضاء و كميات الحركة هو نفسه الفرق بين الفضاء المماسى tangent space (الاسس هى التفاضلات الاتجاهيةبالنسبة للاحداثيات ) والفضاء المماسى المشارك cotangent space (الاسس هى الاحداثيات نفسها )



والله اعلم

تغريد
08-29-2009, 10:58 AM
شكرا لك أخي الكريم لأن جئت لأعتذر عن السؤال فوجدت ردك البليغ جدا

فبارك الله فيك و زادك علما و حكمة

الصادق
08-29-2009, 11:08 PM
مثال:
اذا كانت f دالة فى الاحداثيات المُعممة و كميات الحركة المُعممة والزمن http://latex.codecogs.com/gif.latex?\huge%20f(q,p,t)

برهن ان

http://latex.codecogs.com/gif.latex?\huge%20\frac{df}{dt}=\{f ,H\}_p+\frac{\partial%20f}{\partial %20t}\qquad%20(21)

الحل:
دعنا نحسب تفاضل الدالة f بالنسبة للزمن, ومن اجل هذا الغرض سوف نستخدم قاعدة السلسة (انظر الشرح فى المشاركات السابقة)

http://latex.codecogs.com/gif.latex?\huge%20\frac{df}{dt}=\su m_{i=1}^{N}\frac{\partial%20f}{\par tial%20q_i%20}\dot{q}_i+\frac{\part ial%20f}{\partial%20p_i%20}\dot{p}_ i+\frac{\partial%20f}{\partial%20t}
حيث كتبنا رمز التجميع للدلالة على اننا نجمع جميع القيم الممكنة ل i ولكن يجب على القارئ ان يدرك باننا عندما نكتب حرف مكرر فاننا نعنى التجميع ضمنياً و هذا النوع من الترميز يعرف بقاعدة جمع انشتاين و قد استخدمناه لنوفر على انفسنا مشقة كتابة رمز التجميع مراراً وتكراراً.

الان من معادلات هملتون القانونية نعلم ان
http://latex.codecogs.com/gif.latex?\large%20\dot{q}_i=\frac{ \partial%20H}{\partial%20p_i},%20\q uad%20\dot{p}_i=-\frac{\partial%20H}{\partial%20q_i}

بالتعويض فى العلاقة السابقة نحصل على

http://latex.codecogs.com/gif.latex?\huge%20\frac{df}{dt}=\su m_{i=1}^{N}\frac{\partial%20f}{\par tial%20q_i%20}\frac{\partial%20H}{\ partial%20p_i}-\frac{\partial%20f}{\partial%20p_i% 20}\frac{\partial%20H}{\partial%20q _i}+\frac{\partial%20f}{\partial%20 t}

ولكن من المعادلة (15) نجد ان الحد الذى عليه التجميع ما هو الا تعريف قوسا بوايسون و هكذا نصل الى العلاقة المطلوب برهانها

http://latex.codecogs.com/gif.latex?\huge%20\frac{df}{dt}=\{f ,H\}_p+\frac{\partial%20f}{\partial %20t}\qquad%20(21)

الان اذا كانت http://latex.codecogs.com/gif.latex?\huge%20f(q,p) اى ليست دالة صريحة فى الزمن

فاننا نجد ان

http://latex.codecogs.com/gif.latex?\huge%20\frac{df}{dt}=\{f ,H\}_p%20=0%20\qquad%20(22)

بمعنى انه اذا كانت f تتبادل مع الهملتونيان فان f هى ثابت من ثوابت الحركة اى انها كمية محفوظة

تمرين:
اذا كانت http://latex.codecogs.com/gif.latex?\large%20f_1 و http://latex.codecogs.com/gif.latex?\large%20f_1 ثوابت حركية برهن ان قوس بوايسون لهما http://latex.codecogs.com/gif.latex?\large%20\{f_1,f_2\}_p ايضاً ثابتاً حركياً

تلميح: استخدم خواص اقواس بوايسون


يتبع........

Tyns19
08-30-2009, 12:08 AM
السلام عليكم:


تمرين:
اذا كانت http://latex.codecogs.com/gif.latex?\large%20f_1 و http://latex.codecogs.com/gif.latex?\large%20f_1 ثوابت حركية برهن ان قوس بوايسون لهما http://latex.codecogs.com/gif.latex?\large%20\{f_1,f_2\}_p ايضاً ثابتاً حركياً

تلميح: استخدم خواص اقواس بوايسون

حتى نبرهن أن http://latex.codecogs.com/gif.latex?\large%20\{f_1,f_2\}_p ثابت للحركة يكفي أن نبرهن أن http://www.codecogs.com/eq.latex?\150dpi%20 \{\{f_1,f_2\}_p, H\}_p=0 وبما أنه حسب خاصية ضد التماثلية:


http://www.codecogs.com/eq.latex?\150dpi%20\{\{f_1,f_2\}_p, H\}_p=-\{H,\{f_1,f_2\}_p\}_p

يكفينا إذن أن برهن أن:

http://www.codecogs.com/eq.latex?\150dpi%20 \{H,\{f_1,f_2\}_p\}_p=0

نبدأ الان من متطابقة جاكوبي:

http://www.codecogs.com/eq.latex?\150dpi%20 \{H,\{f_1,f_2\}_p\}_p+\{f_1,\{f_2,H \}_p\}_p+\{f_2,\{H,f_1\}_p\}_p=0

لكن f1 و f2 هما ثوابت للحركة ما يعني أن:

http://www.codecogs.com/eq.latex?\150dpi%20 \{H,f_1\}_p=-\{f_1,H\}_p=0;\qquad \{f_2,H\}_p=0

تصبح متطابقة جاكوبي السابقة كالتالي:

http://www.codecogs.com/eq.latex?\150dpi%20 \{H,\{f_1,f_2\}_p\}_p+\{f_1,0\}_p+\ {f_2,0\}_p=0

لكن بحسب التعريف:

http://www.codecogs.com/eq.latex?\150dpi%20 \{f_1,0\}_p=\{f_2,0\}_p\overset{\ma thrm{def}}{=}0

وبالتعويض في المتطابقة السابقة نجد:

http://www.codecogs.com/eq.latex?\150dpi%20 \{H,\{f_1,f_2\}_p\}_p=0

وهو ما يعني أن http://www.codecogs.com/eq.latex?\150dpi%20\{f_1,f_2\}_p هي أيضا ثابت حركي. وهو المطلوب#.

أتمنى أن أكون قد وفقت.

في انتظار باقي الموضوع.

الصادق
08-30-2009, 12:20 AM
حفظك الله اخى Tyns19 وذادك علماً و حكمة انه حل صحيح وموفق تماماً

تغريد
08-30-2009, 12:24 AM
[COLOR="Green"][U]تمرين:
اذا كانت http://latex.codecogs.com/gif.latex?\large%20f_1 و http://latex.codecogs.com/gif.latex?\large%20f_1 ثوابت حركية برهن ان قوس بوايسون لهما http://latex.codecogs.com/gif.latex?\large%20\{f_1,f_2\}_p ايضاً ثابتاً حركياً







نعلم أن متطابقة جاكوبى

http://latex.codecogs.com/gif.latex?\huge%20\{A,\{B,C\}_p\}_p +\{B,\{C,A\}_p\}_p+\{C,\{A,B\}_p\}_ p=0%20

و عليه فإن


http://latex.codecogs.com/gif.latex?\huge%20\{H,\{f_1,f_2\}_p \}_p+\{f_1,\{f_2,H\}_p\}_p+\{f_2,\{ H,f_1\}_p\}_p=0

http://latex.codecogs.com/gif.latex?\huge%20\{H,\{f_1,f_2\}_p \}_p+\{f_1,0\}_p+\{f_2,0\}_p=0%20


http://latex.codecogs.com/gif.latex?\huge%20\{H,\{f_1,f_2\}_p \}_p+0+0=0%20
و هذا يعني أن http://latex.codecogs.com/gif.latex?\large%20\{f_1,f_2\}_p تتبادل مع الهملتونيان و من ثم فهى ثابت من ثوابت الحركة

الصادق
08-30-2009, 12:31 AM
رائع اختى الكريمة تغريد حل صحيح تماماً

ذادك الله علماً وحكمة

تغريد
08-30-2009, 12:33 AM
لا بأس سبقتني عكاشة
بل و حلك أكثر دقة و تفصيلا
أخي Tyns19
شكرا لك

Tyns19
08-30-2009, 01:10 AM
ما شاء الله عدد كبير من الردود في وقت قياسي، نشاط ملحوظ
كما أن انضمام الأخت تغريد زاد الموضوع رونقا و جمالا
شكرا لك أخت تغريد على الحلول، وان شاء الله سنتابع هذا الموضوع خطوة بخطوة تحت راية أستاذنا الصادق.

الصادق
08-30-2009, 07:30 AM
التحويلات القانونية

التعريف

قلنا ان الاحداثيات فى الفضاء الطورى هى الاحداثيات المُعممة http://latex.codecogs.com/gif.latex?\large%20q_i وكميات الحركة المُعممة http://latex.codecogs.com/gif.latex?\large%20p_i الان نريد ان نوجد تحويلاً من هذه الاحداثيات الى احداثيات جديدة http://latex.codecogs.com/gif.latex?\large%20Q_i و http://latex.codecogs.com/gif.latex?\large%20P_i اى نريد تحويل من النظام القديم الى النظام الجديد http://latex.codecogs.com/gif.latex?\large%20(q_i,p_i)\righta rrow%20(Q_i,P_i) بحيث ان الاحداثيات الجديد تعتمد على الاحداثيات القديمة

http://latex.codecogs.com/gif.latex?\huge%20\\%20Q_i=Q_i(q,p) %20\\%20\\%20P_i=P_i(q,p)

الان طالما ان الهملتونيان فى نظام الاحداثيات القديم يعتمد على الاحداثيات القديمة والزمن http://latex.codecogs.com/gif.latex?\large%20H(q_i,p_i,t) , فان الهملتونيان فى نظام الاحداثيات الجديد يعتمد على الاحداثيات الجديدة و الزمن http://latex.codecogs.com/gif.latex?\large%20K(Q_i,P_i,t) وهكذا فان معادلات هملتون القانونية فى نظام الاحداثيات القديم

http://latex.codecogs.com/gif.latex?\huge%20\dot{q}_i=\frac{\ partial%20H}{\partial%20p_i},%20\qu ad%20\dot{p}_i=-\frac{\partial%20H}{\partial%20q_i}

تقابلها معادلة هملتون القانونية التالية فى نظام الاحداثيات الجديد

http://latex.codecogs.com/gif.latex?\huge%20\dot{Q}_i=\frac{\ partial%20K}{\partial%20P_i},%20\qu ad%20\dot{P}_i=-\frac{\partial%20K}{\partial%20Q_i}

هذا النوع من التحويلات يسمى بالتحويلات القانونية, والفرق بينه وبين التحويلات الاحداثية العامة هو ان التحويلات القانونية تحافظ على شكل معادلة الحركة القانونية

يتبع......

تغريد
08-30-2009, 11:32 AM
ما شاء الله عدد كبير من الردود في وقت قياسي، نشاط ملحوظ
كما أن انضمام الأخت تغريد زاد الموضوع رونقا و جمالا
شكرا لك أخت تغريد على الحلول، وان شاء الله سنتابع هذا الموضوع خطوة بخطوة تحت راية أستاذنا الصادق.

بإذن الله أخي الكريم
كل الشكر لك


أخي الكريم الصادق
آسفة ربما لأن هذه المشاركة لن تساعد في المحافظة على ترتيب الموضوع
و لكن لي بعض الأسئلة قبل الانتقال للتحويلات القانونية

فنحن قد تحدثنا مسبقا عن تجانس الزمان و المكان
و كما هو واضح فإن هذا التجانس كما فهمت يكون بالنسبة للطاقة

و كاننا نتحدث عن مبدأ حفظ الطاقة
و هو مبدأ أستطيع تخيله طالما أن الطاقة تنقسم إلى طاقة الوضع وطاقة الحركة و أن النقص في الأولى تعوضه الزيادة في الثانية

و لكن من أين أتى قانون حفظ كمية الحركة

و آسفة مقدما إذا كان السؤال خارج نطاق الموضوع أو تافها

الصادق
08-31-2009, 01:26 AM
اختى الكريمة تغريد

و لكن من أين أتى قانون حفظ كمية الحركة؟

لو كان الجسم حرأً اى لاتوجد قوى مؤثرة عليه فان كمية الحركة تكون ثابتة http://latex.codecogs.com/gif.latex?\frac{d\vec{p}}{dt}=\vec{ F}=0\rightarrow%20\vec{p}=const وبالتالى فهى كمية محفوظة

والله اعلم

Fatima_a
08-31-2009, 12:34 PM
كل الشكر لكم
شرح رائع جدأ، ومداخلات تُثري الموضوع

حبذا لو تذكرون لنا اسم كتاب بنفس سهولة وسلاسة شرحكم، لكن باللغة الانجليزية

موفقين إن شاء الله

تغريد
08-31-2009, 01:36 PM
شكرا لك أخي الكريم الصادق بارك الله فيك

الصادق
09-04-2009, 08:27 PM
كل الشكر لكم
شرح رائع جدأ، ومداخلات تُثري الموضوع

حبذا لو تذكرون لنا اسم كتاب بنفس سهولة وسلاسة شرحكم، لكن باللغة الانجليزية

موفقين إن شاء الله

شكراً لك Fatima_a على مرورك الكريم

هناك كتابين قد درست منهما الميكانيكا الكلاسيكية وهما


Classical Mechanics by Herbert Goldstein, Charles P. Poole, John L. Safko
Mechanics by L D Landau and E.M. Lifshitz

وهما بالطبع من افضل الكتب فى هذا المجال و يمتازان بالسهولة نسبياً و السلاسة ومن الافضل ان يدرسهما الطالب فى السنة الثانية الجامعية بعد ان يكون قد تعرف على الحسبان متعدد المتغيرات وحساب المتجهات فى السنة الاولى

Tyns19
09-05-2009, 01:23 AM
شكراً لك Fatima_a على مرورك الكريم

هناك كتابين قد درست منهما الميكانيكا الكلاسيكية وهما


Classical Mechanics by Herbert Goldstein, Charles P. Poole, John L. Safko
Mechanics by L D Landau and E.M. Lifshitz

وهما بالطبع من افضل الكتب فى هذا المجال و يمتازان بالسهولة نسبياً و السلاسة ومن الافضل ان يدرسهما الطالب فى السنة الثانية الجامعية بعد ان يكون قد تعرف على الحسبان متعدد المتغيرات وحساب المتجهات فى السنة الاولى

تكليلا لجهود الأخ الصادق، أقدم لكم نسخة الكترونية من الكتابين المذكورين:


Mechanics, By: E M Lifshitz, L D Landau Download (http://rapidshare.com/files/179353429/LL01_djv.rar)
http://ecx.images-amazon.com/images/I/41HJ4Y8N3ML._BO2,204,203,200_PIsitb-sticker-arrow-click,TopRight,35,-76_AA240_SH20_OU01_.jpg






Classical Mechanics, By: Herbert Goldstein, Charles P. Poole, John L. SafkoDownload (http://rs766.rapidshare.com/files/208788659/goldstein.poole.safko_classical.mec hanics.3rd_edition_hosae.rar)
http://images.bestwebbuys.com/muze/books/29/9780201657029.jpg

انشاء الله تكون اضافة موفقة.
في انتظار باقي الموضوع انشاء الله....

الصادق
09-05-2009, 01:53 AM
بالطبع اخى Tyns19 هى اضافة موفقة جداً كما هى جميع مشاركاتك
سوف اتابع الموضوع انشاء الله خلال عطة نهاية الاسبوع

Fatima_a
09-06-2009, 08:19 AM
أعجز عن شكركما
في الحقيقة أنا أدرس حالياً كتاب Goldstein
لكن غير موجود في مكتبات المدينة التي اسكن فيها، وقد قدمت طلب عليه منذ بداية السمستر أي من حوالي أكثر من شهر ولم يصلني حتى الآن

لكن "رمضان كريم"..أخيراً حصلت عليه حتى وإن كان الكترونياً

كل الشكر لكم

Fatima_a
09-06-2009, 08:24 AM
معذرة أستاذي الكريم
فيبدوا أن مشاركاتي في هذا الموضوع ليست في محلها "يعني خربت الموضوع شوي"

فقط وددت أن أسأل عن الرابط للكتاب..على اي برنامج يعمل؟
لأنه لم يعمل معي

أكرر شكري للجميع

Tyns19
09-06-2009, 07:59 PM
فقط وددت أن أسأل عن الرابط للكتاب..على اي برنامج يعمل؟
لأنه لم يعمل معي
يعمل بواسطة مشغل djvu تفضلي بالتحميل من هنا (http://sourceforge.net/projects/windjview/files/WinDjView/1.0.3/WinDjView-1.0.3-Setup.exe/download)

مبروك التوقيع الجديد أخي الصادق، سرني كثيرا استبدال العبارة السابقة.

الصادق
09-07-2009, 03:33 AM
التحويلات القانونية و مولدات التماثل

قلنا ان التحويلات القانونية فى ميكانيكا هملتون تُعرف بانها التحويلات من نظام الاحداثيات http://latex.codecogs.com/gif.latex?\large%20(q,p) الى نظام احداثيات جديد http://latex.codecogs.com/gif.latex?\large%20(Q,P) بحيث يحافظ التحويل على شكل معادلات هملتون. وطالما ان التحويلات القانونية تحافظ على شكل معادلات هماتون القانونية فانها ايضاً تحافظ على صيغة اقواس بوايسون

الان دعنا نُعرف اقواس بوليسون للدالتين A و B فى نظام الاحداثيات http://latex.codecogs.com/gif.latex?\large%20(q,p) ( من اجل التبسيط سوف اهتم فقط بحالة درجة الحرية الواحدة)

http://latex.codecogs.com/gif.latex?\huge%20\{A,B\}_{q,p}=\fr ac{\partial%20A}{\partial%20q}\frac {\partial%20B}{\partial%20p}-\frac{\partial%20B}{\partial%20q}\f rac{\partial%20A}{\partial%20p}\qqu ad(21)

وهكذا فاننا نستطيع كتابة التفاضلات التى تظهر فى المعادلة (21) بدلالة الاحداثيات الجديدة http://latex.codecogs.com/gif.latex?\large%20(Q,P) على النحو التالى

http://latex.codecogs.com/gif.latex?\huge%20\\%20\frac{\parti al%20A}{\partial%20q}=\frac{\partia l%20A}{\partial%20Q}\frac{\partial% 20Q}{\partial%20q}+\frac{\partial%2 0A}{\partial%20P}\frac{\partial%20P }{\partial%20q}%20\\%20\\%20\\%20\f rac{\partial%20A}{\partial%20p}=\fr ac{\partial%20A}{\partial%20Q}\frac {\partial%20Q}{\partial%20p}+\frac{ \partial%20A}{\partial%20P}\frac{\p artial%20P}{\partial%20p}%20\\%20\\ %20\\%20\frac{\partial%20B}{\partia l%20q}=\frac{\partial%20B}{\partial %20Q}\frac{\partial%20Q}{\partial%2 0q}+\frac{\partial%20B}{\partial%20 P}\frac{\partial%20P}{\partial%20q} %20\\%20\\%20\\%20\frac{\partial%20 B}{\partial%20p}=\frac{\partial%20B }{\partial%20Q}\frac{\partial%20Q}{ \partial%20p}+\frac{\partial%20B}{\ partial%20P}\frac{\partial%20P}{\pa rtial%20p}%20\qquad%20(22)


وهكذا بالتعويض فى فى المعادلة (21) سوف نحصل على

http://latex.codecogs.com/gif.latex?\large%20\\%20\{A,B\}_{q, p}=\left(\frac{\partial%20A}{\parti al%20Q}\frac{\partial%20Q}{\partial %20q}+\frac{\partial%20A}{\partial% 20P}\frac{\partial%20P}{\partial%20 q}\right)\left(%20\frac{\partial%20 B}{\partial%20Q}\frac{\partial%20Q} {\partial%20p}+\frac{\partial%20B}{ \partial%20P}\frac{\partial%20P}{\p artial%20p}\right%20)-\left(\frac{\partial%20B}{\partial% 20Q}\frac{\partial%20Q}{\partial%20 q}+\frac{\partial%20B}{\partial%20P }\frac{\partial%20P}{\partial%20q}\ right)\left(%20\frac{\partial%20A}{ \partial%20Q}\frac{\partial%20Q}{\p artial%20p}+\frac{\partial%20A}{\pa rtial%20P}\frac{\partial%20P}{\part ial%20p}\right%20)\\%20\\%20\\%20\{ A,B\}_{q,p}=\left(\frac{\partial%20 A}{\partial%20Q}%20\frac{\partial%2 0B}{\partial%20P}-\frac{\partial%20B}{\partial%20Q}%2 0\frac{\partial%20A}{\partial%20P}\ right%20)\frac{\partial%20Q}{\parti al%20q}\frac{\partial%20P}{\partial %20p}-\left(\frac{\partial%20A}{\partial% 20Q}%20\frac{\partial%20B}{\partial %20P}-\frac{\partial%20B}{\partial%20Q}%2 0\frac{\partial%20A}{\partial%20P}\ right%20)\frac{\partial%20P}{\parti al%20q}\frac{\partial%20Q}{\partial %20p}



الان تلاحظ ان القوس فى الطرف الايمن فى المعادلة الاخيرة ماهو الا قوس بوايسون للدوال A و B فى نظام الاحداثيات الجديد Q,Pاى ان

http://latex.codecogs.com/gif.latex?\large%20\\%20\{A,B\}_{q, p}=\{A,B\}_{Q,P}\frac{\partial%20Q} {\partial%20q}\frac{\partial%20P}{\ partial%20p}-\{A,B\}_{Q,P}\frac{\partial%20P}{\p artial%20q}\frac{\partial%20Q}{\par tial%20p}%20\\%20\\%20\\%20\{A,B\}_ {q,p}=\{A,B\}_{Q,P}\left(\frac{\par tial%20Q}{\partial%20q}\frac{\parti al%20P}{\partial%20p}-\frac{\partial%20P}{\partial%20q}\f rac{\partial%20Q}{\partial%20p}%20\ right%20)


و هكذا لكى يحافظ التحويل القانونى على شكل اقواس بوايسون

http://latex.codecogs.com/gif.latex?\large%20\{A,B\}_{q,p}=\{ A,B\}_{Q,P}

فيجب على الحد بين القوسين ان يساوى 1 اى ان

http://latex.codecogs.com/gif.latex?\large%20\left(\frac{\par tial%20Q}{\partial%20q}\frac{\parti al%20P}{\partial%20p}-\frac{\partial%20P}{\partial%20q}\f rac{\partial%20Q}{\partial%20p}%20\ right%20)=1

لاحظ ان هذا الحد بين القوسين ما هو الا تعريف اقواس بوايسون للاحداثيات الجديدة بدلالة الاحداثيات القديمة

http://latex.codecogs.com/gif.latex?\huge%20\{Q,P\}_{q,p}=%20 \frac{\partial%20Q}{\partial%20q}\f rac{\partial%20P}{\partial%20p}-\frac{\partial%20P}{\partial%20q}\f rac{\partial%20Q}{\partial%20p}%20\ right%20)=1%20\qquad%20(23)

الان التعميم الى حالة N درجة حرية سهل و مباشر و يعطى العلاقة التالية

http://latex.codecogs.com/gif.latex?\huge%20\{Q_i,P_j\}_{q,p} =%20\frac{\partial%20Q_i}{\partial% 20q_k}\frac{\partial%20P_j}{\partia l%20p_k}-\frac{\partial%20P_j}{\partial%20q_ k}\frac{\partial%20Q_i}{\partial%20 p_k}%20\right%20)=\delta_{ij}%20\qq uad%20(24)


يتبع.............

محمد ابوزيد
09-07-2009, 04:01 AM
اشكرك اخى الصادق ومحاضرات لا تقدر بمال

استمر حسبك الله معك

اخوكم / محمد ابوزيد

الصادق
09-07-2009, 05:04 AM
شكراً لك اخى محمد ابو زيد اسعدتنى بمرورك الكريم و كلماتك الطيبات

الصادق
09-07-2009, 06:31 AM
التحويلات القانونية متناهية الصغر

افترض تحويل قانونى متناهى الصغر اى تحويل بحيث ان الاحداثيات الجديدة هى عبارة عن تغير طفيف على الاحداثيات القديمة

http://latex.codecogs.com/gif.latex?\huge%20\\%20Q=q+\delta%2 0q%20=q+\epsilon%20f(q,p)\\%20\\%20 P=p+\delta%20p%20=q+\epsilon%20h(q, p)%20\qquad%20(25)

بحيث ان http://latex.codecogs.com/gif.latex?\large%20\epsilon هى معامل متناهى الصغر يؤول الى الصفر
الان بالتعويض فى المعادلة (23)

http://latex.codecogs.com/gif.latex?\huge%20\{Q,P\}_{q,p}=%20 \frac{\partial%20Q}{\partial%20q}\f rac{\partial%20P}{\partial%20p}-\frac{\partial%20P}{\partial%20q}\f rac{\partial%20Q}{\partial%20p}%20\ right%20)=1%20\qquad%20(23)

نحصل على

http://latex.codecogs.com/gif.latex?\large%20\\%20\left(1+\ep silon%20\frac{\partial%20f}{\partia l%20q}\right)\left(1+\epsilon%20\fr ac{\partial%20h}{\partial%20p}\righ t)-\left(\epsilon%20\frac{\partial%20h }{\partial%20q}\right)\left(\epsilo n%20\frac{\partial%20f}{\partial%20 p}\right)=1%20\\%20\\%201+\epsilon% 20\left(\frac{\partial%20f}{\partia l%20q}+\frac{\partial%20h}{\partial %20p}%20\right%20)+\epsilon^2\left( \frac{\partial%20f}{\partial%20q}\f rac{\partial%20h}{\partial%20p}-\frac{\partial%20h}{\partial%20q}\f rac{\partial%20f}{\partial%20p}\rig ht%20)=1


و الان طالما ان http://latex.codecogs.com/gif.latex?\large%20\epsilon متناهية الصغر فاننا سوف نهمل الحد الحد الذى يتضمن المربع

http://latex.codecogs.com/gif.latex?\large%201+\epsilon%20\le ft(\frac{\partial%20f}{\partial%20q }+\frac{\partial%20h}{\partial%20p} %20\right%20)=1

اى ان

http://latex.codecogs.com/gif.latex?\huge%20\frac{\partial%20 f}{\partial%20q}=-\frac{\partial%20h}{\partial%20p}


وهذه المعادلة تقود الى ان الدوال f و h يجب ان تحقق العلاقات التالية

http://latex.codecogs.com/gif.latex?\huge%20f=\frac{\partial% 20G}{\partial%20p};%20\qquad%20h=-\frac{\partial%20G}{\partial%20q}%2 0\qquad%20(26)

والدالة G تسمى بالدالة المولدة لتحويل القانونى

وهذا بالتعويض بالتعويض فى المعادلة (25) نحصل على

http://latex.codecogs.com/gif.latex?\huge%20\\Q=q+\epsilon%20 \frac{\partial%20G}{\partial%20p}\\ %20\\%20P=p-\epsilon%20\frac{\partial%20G}{\par tial%20q}

والان من الواضح ان التعميم لحالة N درجة حرية يعطى الصورة التالية

http://latex.codecogs.com/gif.latex?\huge%20\\Q_i=q_i+\delta% 20q_i=q_i+\epsilon%20\frac{\partial %20G}{\partial%20p_i}\\%20\\%20P_i= p_i+\delta%20p_i%20=p_i-\epsilon%20\frac{\partial%20G}{\par tial%20q_i}\qquad%20(27)

وهكذا فان التغيرات فى الاحداثيات تعطى ب

http://latex.codecogs.com/gif.latex?\huge%20\\%20\delta%20q_i =\epsilon%20\frac{\partial%20G}{\pa rtial%20p_i};%20\qquad%20\delta%20p _i%20=-\epsilon%20\frac{\partial%20G}{\par tial%20q_i}\qquad%20(28)

يتبع......

الصادق
09-08-2009, 03:32 AM
تمارين

1-برهن ان التحويل

http://latex.codecogs.com/gif.latex?\large%20\\Q=-p%20\\%20\\%20P=q+\alpha%20p^2

(حيث الفا ثابت) يمثل تحويلاً قانونياً

2-برهن ان التحويل

http://latex.codecogs.com/gif.latex?\large%20\\Q_1=\frac{1}{\ sqrt{2}}(q_1+\frac{p_2}{m\omega});% 20\qquad%20P_1=\frac{1}{\sqrt{2}}(p _1-m\omega%20q_2)%20\\%20\\%20\\Q_2=\f rac{1}{\sqrt{2}}(q_1-\frac{p_2}{m\omega});%20\qquad%20P_ 2=\frac{1}{\sqrt{2}}(p_1+m\omega%20 q_2)

(حيث m اوميقا هى ثوابت) يمثل تحويلاً قانونياً

3- برهن ان

http://latex.codecogs.com/gif.latex?\large%20H(q+\epsilon%20\ frac{\partial%20G}{\partial%20p},p-\epsilon%20\frac{\partial%20G}{\par tial%20q},t)=H(q,p,t)+\epsilon%20\f rac{\partial%20G}{\partial%20t}

ماذا تستنتج فى حال كانت الدالة المولدة للتحويل G لاتعتمد على الزمن اعتماداً صريحاً؟

الصادق
09-08-2009, 05:38 PM
نظرية نويزر

افترض تحويلاً قانونياً متناهى الصغر تولده دالة G. الان نجد ان التغير فى دالة هملتون يُعطى ب

http://latex.codecogs.com/gif.latex?\huge%20\delta%20H=\frac{ \partial%20H}{\partial%20q_i}\delta %20q_i+\frac{\partial%20H}{\partial %20p_i}\delta%20p_i

ولكن قد برهنا سابقاً (المعادلات (28)) ان

http://latex.codecogs.com/gif.latex?\huge%20\\%20\delta%20q_i =\epsilon%20\frac{\partial%20G}{\pa rtial%20p_i};%20\qquad%20\delta%20p _i%20=-\epsilon%20\frac{\partial%20G}{\par tial%20q_i}\qquad%20(28)

وبالتعويض فى المعادلة اعلاه نجد ان التغير فى دالة هملتون يعطى ب

http://latex.codecogs.com/gif.latex?\huge%20\\%20\delta%20H=\ epsilon\frac{\partial%20H}{\partial %20q_i}\frac{\partial%20G}{\partial %20p_i}-\epsilon\frac{\partial%20H}{\partia l%20p_i}\frac{\partial%20G}{\partia l%20q_i}%20\\%20\\%20\delta%20H%20= \epsilon\{H,G\}\qquad%20(29)

وعندما تكون G تمثل تماثلاً للهملتونيان (دالة هملتون) اى ان http://latex.codecogs.com/gif.latex?\delta%20H=0 فان G يجب ان تتبادل من خلال قوسا بوايسون مع الهملتونيان

http://latex.codecogs.com/gif.latex?\huge%20\{H,G\}=0

وهكذا طالما ان http://latex.codecogs.com/gif.latex?\large%20\frac{\mathrm{d} %20G%20}{\mathrm{d}%20t}=\dot{G}=\{ H,G\} فان G كمية محفوظة.

نلخص ماسبق بالقول التالى " ان اى تماثل للمنظومة الفيزيائية تحت تحويل تولده دالة G فان هناك كمية محفوظة وهى G نفسها

التماثل تحت التحويل الانتقالى الزمنى

الانتقال الزمنى هو

http://latex.codecogs.com/gif.latex?\large%20t%27=t+\delta%20 t

اى ان المعامل المتناهى الصغر للتحويل هو http://latex.codecogs.com/gif.latex?\large%20\epsilon%20=\del ta%20t وبالتعويض فى المعادلة (29) نحصل على


http://latex.codecogs.com/gif.latex?\huge%20\delta%20H=\delta %20t%20\{H,G\}=\delta%20t%20\frac{\ mathrm{d}%20G}{\mathrm{d}%20t}=\del ta%20G

وبالتكامل نحصل على G=H اى ان الدالة المولدة للانتقال الزمنى هى الهملتونيان (الطاقة الكلية للمنظومة)

الان اذا كانت المنظومة متماثلة تحت الانتقال الزمنى فان الطاقة تكون محفوظة اى ان

http://latex.codecogs.com/gif.latex?\huge%20\delta%20H=\delta %20t%20\{H,H\}=0

حيث عوضنا الدالة المولدة للانتقال الزمنى G=H لاحظ انه طالما ان اى دالة مهما كانت فهى تتبادل من خلال قوسا بوايسون مع نفسها فان الطاقة الكلية الهملتونيان تكون دائماً محفوظة

التماثل تحت التحويل الانتقالى المكانى

الانتقال المكانى هو

http://latex.codecogs.com/gif.latex?\huge%20q%27=q+\delta%20q

اى ان المعامل المتناهى الصغر للتحويل هو http://latex.codecogs.com/gif.latex?\large%20\epsilon%20=\del ta%20q وبالتعويض فى المعادلة الاولى من (28) نحصل على

http://latex.codecogs.com/gif.latex?\huge%20\delta%20q=\delta %20q%20\frac{\partial%20G}{\partial %20p}\Rightarrow%20\frac{\partial%2 0G}{\partial%20p}=1

وباجراء التكامل نحصل على G=p اى ان الدالة المولدة للانتقال المكانى هى كمية الحركة


الان اذا كانت المنظومة متماثلة تحت الانتقال المكانى فان كمية الحركة تكون محفوظة. ولكى ترى ذلك عوض فى المعادلة الثانية من (28)

http://latex.codecogs.com/gif.latex?\huge%20\delta%20p=-\delta%20q%20\frac{\partial%20p}{\p artial%20q}=0

حيث عوضنا الدالة المولدة للانتقال المكانى G=p.


يتبع....

الصادق
09-09-2009, 03:21 AM
التماثل تحت التحويل الدورانى

تمهيد

دعنا من اجل التبسيط نفترض فضاء ثلاثى الابعاد x,y,z و نقوم بتدوير المحاور x,y حول المحور z بزاوية http://latex.codecogs.com/gif.latex?\large%20\theta كما هو موضح بالرسم

http://up1.arb-up.com/files/arb-up-2009-8/cQB52696.jpg


ومن الرسم نلاحظ ان المركبات بعد الدوران تعطى ب

http://latex.codecogs.com/gif.latex?\huge%20\\%20x%27=x\cos\t heta%20-y\sin\theta%20\\\\%20y%27=x\sin\the ta%20+y\cos%20\theta%20\qquad(30\;a )

وبالطبع اذا تم تدوير مركبات كمية الحركة ثلاثية الابعاد http://latex.codecogs.com/gif.latex?\large%20p_x,%20p_y,%20p_ z حول المحور http://latex.codecogs.com/gif.latex?\large%20p_z فاننا سوف نحصل على

http://latex.codecogs.com/gif.latex?\huge%20\\%20p%27_{x}=p_x \cos\theta%20-p_y\sin\theta%20\\\\%20p%27_y=p_x\s in\theta%20+p_y\cos%20\theta%20\qqu ad(30\;b)

الان افترض تحويلاً دورانياً متناهى الصغر اى دوران بزاوية صغيرة http://latex.codecogs.com/gif.latex?\large%20\delta%20\theta تؤول الى الصفر, وطالما ان الزاوية صغيرة فاننا نقرب جيب و جيب تمام الزاوية ب

http://latex.codecogs.com/gif.latex?\huge%20\cos\delta%20\the ta=1,%20\qquad%20\sin%20\theta=\del ta%20\theta

و بالتعويض فى العلاقات (30a) و(30b) نحصل على التحويل الدورانى متناهى الصغر
http://latex.codecogs.com/gif.latex?\huge%20\\%20x%27=x%20-y\delta\theta%20\\\\%20y%27=x\delta \theta%20+y%20\qquad(31\;a)

http://latex.codecogs.com/gif.latex?\huge%20\\%20p%27_{x}=p_x %20-p_y\delta\theta%20\\\\%20p%27_y=p_x \delta\theta%20+p_y%20\qquad(31\;b)

وهكذا فان التغير فى المحاور الاحداثية ومركبات كميات الحركة يعطى ب

http://latex.codecogs.com/gif.latex?\huge%20\\\delta%20x=%20x %27-x%20=-y\delta\theta%20\\\\%20\delta%20y=y %27-y=x\delta\theta\qquad(32\;a)

http://latex.codecogs.com/gif.latex?\huge%20\\\delta%20p_x=%2 0p_x%27-p_x%20=-y\delta\theta%20\\\\%20\delta%20p_y =p_y%27-p_y=p_x\delta\theta\qquad(32\;b)

الان لنعود مرة اخرى الى متغيرات فضاء الطور و نستبدل x,y ب q_1,q_2 ونستبدل p_x,p_y ب p_1,p_2 و هكذا نستطيع كتابة التحويل الدورانى متناهى الصغر فى فضاء الطور على النحو التالى

http://latex.codecogs.com/gif.latex?\huge%20\\\delta%20q_1%20 =-q_2\delta\theta%20\\\\%20\delta%20q _2=q_1\delta\theta\qquad(33\;a)\\%2 0\\%20\\\delta%20p_1%20=-p_2\delta\theta%20\\\\%20\delta%20p _2=p_1\delta\theta\qquad(33\;b)

يتبع......

الصادق
09-09-2009, 05:20 AM
الدوران يُعطى ب http://latex.codecogs.com/gif.latex?\large%20\theta%27=\theta +\delta%20\theta وعليه فان معامل التحويل الدورانى متناهى الصغر هو http://latex.codecogs.com/gif.latex?\large%20\epsilon%20=\del ta%20\theta وبالتعويض فى المعادلات (28) نجد ان

http://latex.codecogs.com/gif.latex?\huge%20\\%20\delta%20q_i =\delta \theta%20\frac{\partial%20G}{\parti al%20p_i};%20\qquad%20\delta%20p_i% 20=-\delta \theta%20\frac{\partial%20G}{\parti al%20q_i}\qquad%20

والان نلاحظ من المعادلات (33a) و (33b) انه لدينا مركبتان للحداثى المكانى و مركبتان لكمية الحركة لذا فان i تأخذ القيم 1و2

http://latex.codecogs.com/gif.latex?\huge%20\\%20\delta%20q_1 =\delta%20\theta%20\frac{\partial%2 0G}{\partial%20p_1}\\%20\\%20\\%20\ delta%20q_2=\delta%20\theta%20\frac {\partial%20G}{\partial%20p_2}%20\\ %20\\%20\\%20\delta%20p_1=-\delta%20\theta%20\frac{\partial%20 G}{\partial%20q_1}\\%20\\%20\\%20\d elta%20p_2=-\delta%20\theta%20\frac{\partial%20 G}{\partial%20q_2}

وبتعويض التغيرات http://latex.codecogs.com/gif.latex?\large%20\delta%20q_1,%20 \;%20\delta%20q_2,%20\;\delta%20p_1 ,%20\;\delta%20p_2 من المعادلات (33a) و (33b) نحصل على

http://latex.codecogs.com/gif.latex?\large%20\\-q_2%20\delta%20\theta%20&=&\delta%20\theta%20\frac{\partial%20 G}{\partial%20p_1}\quuad%20\Rightar row\;%20\frac{\partial%20G}{\partia l%20p_1}=-q_2%20\\%20\\%20\\%20q_1%20\delta%2 0\theta%20&=&\delta%20\theta%20\frac{\partial%20 G}{\partial%20p_2}\quuad%20\Rightar row\;%20\frac{\partial%20G}{\partia l%20p_2}=q_1%20\\%20\\%20\\%20-p_2%20\delta%20\theta=-\delta%20\theta%20\frac{\partial%20 G}{\partial%20q_1}\quuad%20\Rightar row\;%20\frac{\partial%20G}{\partia l%20q_1}=p_2\\%20\\%20\\%20p_1\delt a%20\theta=-\delta%20\theta%20\frac{\partial%20 G}{\partial%20q_2}\quuad%20\Rightar row\;%20\frac{\partial%20G}{\partia l%20q_2}=-p_1%20\qquad%20(34)

الان هذه المعادلات تقول ان الدالة G المولدة للدوران تعتمد على http://latex.codecogs.com/gif.latex?\large%20q_1,\;%20q_2,\;% 20p_1,\;%20p_2 و لذلك فان تكامل المعادلة فى السطر الاول من (34) يعطى http://latex.codecogs.com/gif.latex?\large%20G=-q_2p_1+f(q_1,p_2) حيث f تمثل ثابت التكامل الجزئى. والان اذا فاضلنا G بالنسبة ل http://latex.codecogs.com/gif.latex?\large%20p_2 و قارنا الناتج مع المعادلة فى السطر الثانى من (34) سوف نجد ان http://latex.codecogs.com/gif.latex?\large%20\frac{\partial%2 0f}{\partial%20p_2}=q_1 اى ان http://latex.codecogs.com/gif.latex?\large%20f(q_1,p_2)=q_1p_ 2 و هكذا فان الدالة المولدة للتحويل الدورانى هى

http://latex.codecogs.com/gif.latex?\huge%20G=q_1p_2-q_2p_1


ولكن هذه الدالة ما هى الا كمية الحركة الزاوية فى اتجاه المحور z وهكذا فان الدوران حول محور z تولده كمية الحركة الزاوية فى اتجاه المحور z وعموماً نجد ان الدوران فى اتجاه متجه الوحدة n تولده كمية الحركة الزاوية فى ذلك الاتجاه اى ان

http://latex.codecogs.com/gif.latex?\huge%20G=\mathbf{J}.\mat hbf{n}

تمرين: اذا كانت المنظومة متماثلة تحت التحويل الدورانى, برهن ان كمية الحركة الزاوية تكون محفوظة

يتبع.............

الصادق
09-09-2009, 05:22 AM
يعمل بواسطة مشغل djvu تفضلي بالتحميل من هنا (http://sourceforge.net/projects/windjview/files/WinDjView/1.0.3/WinDjView-1.0.3-Setup.exe/download)

مبروك التوقيع الجديد أخي الصادق، سرني كثيرا استبدال العبارة السابقة.

بارك الله فيك واسعدك دوماً اخى Tyns19

تغريد
09-09-2009, 09:31 PM
تمارين

1-برهن ان التحويل

http://latex.codecogs.com/gif.latex?\large%20\\Q=-p%20\\%20\\%20P=q+\alpha%20p^2

(حيث الفا ثابت) يمثل تحويلاً قانونياً



سأحاول بداية في السؤال الأول لضيق الوقت و لأني لا زلت أجد صعوبة في كتابة المعادلات


نظرا لأن


http://latex.codecogs.com/gif.latex?\large%20\\Q=-p%20\\%20\\%20P=q+\alpha%20p^2


فإنه بالاستعانة بمعادلات هاملتون للحركة ينتج أن
http://latex.codecogs.com/gif.latex?\.{Q}_{i}=-\.{p}_{i}=-(-\frac{\partial H}{\partial q_{i}})=\frac{\partial H}{\partial q_{i}}

و لكن بإيجاد المشتقات الجزئية للصيغ المعطاة و التعويض عنها ينتج أن

http://latex.codecogs.com/gif.latex?\frac{\partial H}{\partial q_{i}}=\frac{\partial H}{\partial Q_{i}}\frac{\partial Q_{i}}{\partial q_{i}}+ \frac{\partial H}{\partial P_{i}}\frac{\partial P_{i}}{\partial q_{i}}=\frac{\partial H}{\partial P_{i}}

و عليه فإن

---------------------

http://latex.codecogs.com/gif.latex?\.{Q}_{i}=\frac{\partial H}{\partial P_{i}}

---------------------
و بالمثل يمكننا الوصول إلى أن

http://latex.codecogs.com/gif.latex?\.{P}_{i}=\.{q}_{i}+ 2\alpha p_{i}\.{p}_{i} =\frac{\partial H}{\partial p_{i}}-2\alpha p_{i}\frac{\partial H}{\partial q_{i}}

و من ثم

http://latex.codecogs.com/gif.latex?\frac{\partial H}{\partial p_{i}}=\frac{\partial H}{\partial Q_{i}}\frac{\partial Q_{i}}{\partial p_{i}}+ \frac{\partial H}{\partial P_{i}}\frac{\partial P_{i}}{\partial p_{i}}=-\frac{\partial H}{\partial Q_{i}}+2\alpha p\frac{\partial H}{\partial P_{i}}

و بالتعويض ينتج أن
---------------------

http://latex.codecogs.com/gif.latex?\.{P}_{i}=-\frac{\partial H}{\partial Q_{i}}+2\alpha p\frac{\partial H}{\partial P_{i}}-2\alpha p_{i}\frac{\partial H}{\partial P_{i}}=-\frac{\partial H}{\partial Q_{i}}

---------------------

و الله أعلم


نلاحظ أن معادلات هاملتون حافظت على شكلها و نلاحظ ايضا أن
نلاحظ أن الهاملتونيان بقي كما هو في النظامين السابق و اللاحق
و لا أعلم هل سيكون الأمر كذلك في باقي الأسئلة أم لا ؟؟؟

الصادق
09-09-2009, 10:09 PM
سأحاول بداية في السؤال الأول لضيق الوقت و لأني لا زلت أجد صعوبة في كتابة المعادلات


نظرا لأن


http://latex.codecogs.com/gif.latex?\large%20\\q=-p%20\\%20\\%20p=q+\alpha%20p^2


فإنه بالاستعانة بمعادلات هاملتون للحركة ينتج أن
http://latex.codecogs.com/gif.latex?\.{q}_{i}=-\.{p}_{i}=-(-\frac{\partial h}{\partial q_{i}})=\frac{\partial h}{\partial q_{i}}

و لكن بإيجاد المشتقات الجزئية للصيغ المعطاة و التعويض عنها ينتج أن

http://latex.codecogs.com/gif.latex?\frac{\partial h}{\partial q_{i}}=\frac{\partial h}{\partial q_{i}}\frac{\partial q_{i}}{\partial q_{i}}+ \frac{\partial h}{\partial p_{i}}\frac{\partial p_{i}}{\partial q_{i}}=\frac{\partial h}{\partial p_{i}}

و عليه فإن

---------------------

http://latex.codecogs.com/gif.latex?\.{q}_{i}=\frac{\partial h}{\partial p_{i}}

---------------------
و بالمثل يمكننا الوصول إلى أن

http://latex.codecogs.com/gif.latex?\.{p}_{i}=\.{q}_{i}+ 2\alpha p_{i}\.{p}_{i} =\frac{\partial h}{\partial p_{i}}-2\alpha p_{i}\frac{\partial h}{\partial q_{i}}

و من ثم

http://latex.codecogs.com/gif.latex?\frac{\partial h}{\partial p_{i}}=\frac{\partial h}{\partial q_{i}}\frac{\partial q_{i}}{\partial p_{i}}+ \frac{\partial h}{\partial p_{i}}\frac{\partial p_{i}}{\partial p_{i}}=-\frac{\partial h}{\partial q_{i}}+2\alpha p\frac{\partial h}{\partial p_{i}}

و بالتعويض ينتج أن
---------------------

http://latex.codecogs.com/gif.latex?\.{p}_{i}=-\frac{\partial h}{\partial q_{i}}+2\alpha p\frac{\partial h}{\partial p_{i}}-2\alpha p_{i}\frac{\partial h}{\partial p_{i}}=-\frac{\partial h}{\partial q_{i}}

---------------------

و الله أعلم


نلاحظ أن معادلات هاملتون حافظت على شكلها و نلاحظ ايضا أن
نلاحظ أن الهاملتونيان بقي كما هو في النظامين السابق و اللاحق
و لا أعلم هل سيكون الأمر كذلك في باقي الأسئلة أم لا ؟؟؟

حياك الله اختى الكريمة تغريد
حلك صحيح و جميل جداً

حفظك الله و زادك علماً

تغريد
09-10-2009, 09:57 PM
\.{G}=0
تمارين

1-برهن ان التحويل

http://latex.codecogs.com/gif.latex?\large%20\\Q=-p%20\\%20\\%20P=q+\alpha%20p^2

(حيث الفا ثابت) يمثل تحويلاً قانونياً

2-برهن ان التحويل

http://latex.codecogs.com/gif.latex?\large%20\\Q_1=\frac{1}{\ sqrt{2}}(q_1+\frac{p_1}{m\omega});% 20\qquad%20P_1=\frac{1}{\sqrt{2}}(p _1-m\omega%20q_2)%20\\%20\\%20\\Q_2=\f rac{1}{\sqrt{2}}(q_1-\frac{p_1}{m\omega});%20\qquad%20P_ 1=\frac{1}{\sqrt{2}}(p_1+m\omega%20 q_2)

(حيث m اوميقا هى ثوابت) يمثل تحويلاً قانونياً

3- برهن ان

http://latex.codecogs.com/gif.latex?\large%20H(q+\epsilon%20\ frac{\partial%20G}{\partial%20p},p-\epsilon%20\frac{\partial%20G}{\par tial%20q},t)=H(q,p,t)+\epsilon%20\f rac{\partial%20G}{\partial%20t}

ماذا تستنتج فى حال كانت الدالة المولدة للتحويل G لاتعتمد على الزمن اعتماداً صريحاً؟

بحمد الله حللنا السؤال الأول و السؤال الثاني يحل بنفس الطريقة و لكني سأتركه للاخوة و الأخوات
أما السؤال الثالث فغلبني حقيقة
لأني ربما ربما ل 15 سنة لم أتعامل مع التفاضلات الجزئية كما أنني لم أثق يوما بفهمي لها
لذا حاولت أن أستشف من مشاركات أخي الصادق التالية للسؤال الحل
بالإضافة إلى كون الجواب النهائي معلوم

لذا جاء حلي كالتالي
و أرجو ألا أكون قد أخطأت

يمكن كتابة التغير في الهاملتونيان نتيجة التحويلات المعطاة كالتالي

http://latex.codecogs.com/gif.latex?\delta H=\frac{\partial H}{\partial q_i}\delta q_i+\frac{\partial H}{\partial p_i}\delta p_i+\frac{\partial H}{\partial t}\delta t

و لكن نعلم أن

http://latex.codecogs.com/gif.latex?\delta q_i=\epsilon \frac{\partial G}{\partial p_i}; \qquad \delta p_i =-\epsilon \frac{\partial G}{\partial q_i};\qquad \delta t=0



http://latex.codecogs.com/gif.latex?\Rightarrow \delta H=\epsilon\frac{\partial H}{\partial q_i}\frac{\partial G}{\partial p_i}-\epsilon\frac{\partial H}{\partial p_i}\frac{\partial G}{\partial q_i}


http://latex.codecogs.com/gif.latex?\delta H=- \epsilon \.{p}\frac{\partial G}{\partial p_i}-\epsilon \.{q} \frac{\partial G}{\partial q_i}=- \epsilon ( \frac{\partial G}{\partial p_i}\.{p}+ \frac{\partial G}{\partial q_i}\.{q})


و نظرا لأن

http://latex.codecogs.com/gif.latex?\delta t=0

مما يعني أن


http://latex.codecogs.com/gif.latex?\.{G}=0


http://latex.codecogs.com/gif.latex?\Rightarrow 0=\.{G}= \frac{\partial G}{\partial p_i}\.{p}+ \frac{\partial G}{\partial q_i}\.{q}+\frac{\partial G}{\partial t}


http://latex.codecogs.com/gif.latex?\Rightarrow \frac{\partial G}{\partial t}=- (\frac{\partial G}{\partial p_i}\.{p}+ \frac{\partial G}{\partial q_i}\.{q})

http://latex.codecogs.com/gif.latex?\Rightarrow \Rightarrow \delta H=- \epsilon ( \frac{\partial G}{\partial p_i}\.{p}+ \frac{\partial G}{\partial q_i}\.{q})= \epsilon \frac{\partial G}{\partial t}



http://latex.codecogs.com/gif.latex? \Rightarrow H(q+\epsilon\frac{\partial G}{\partial p},p-\epsilon\frac{\partial G}{\partial q},t)=H(q,p,t)+\epsilon \frac{\partial G}{\partial t}


مما يعني أنه إذا كانت الدالة المولدة للتحويل G لاتعتمد على الزمن اعتماداً صريحاً فإن الهماملتونيان تكون محفوظة

و الله تعالى أعلم

الصادق
09-12-2009, 08:03 AM
شكراً اختى الكريمة تغريد
حل صحيح
ذادك الله علماً وفضلاً

تغريد
09-12-2009, 03:15 PM
و لك خالص الشكر اخي الكريم
صدقا لقد أعدت ثقتي بما كنت أسميه التفاضل و مشتقاته و الذي أعتقد أنه (فيما عدا التفاضل نفسه) درس لنا بطريقة جعلتني لا أثق به كعلم رغم أني كنت أحرز فيه درجات عالية

و لكني لم أشعر يوما أني متمكنة بسبب بعض الإشكالات في الفهم و لكن إذا كان حلي صحيحا

(و خاصة الخطوة المتعلقة بالتعبير عن تفاضل الدالة G بالنسبة للزمن )

فهذا يعني أن بعض المشاكل لوكنت تابعتها لما كانت لدي هذه الحساسية مع الرياضيات التطبيقية

و خاصة الخطوة المتعلقة بالتعبير عن تفاضل الدالة G بالنسبة للزمن


فكل الشكر لك أخي الفاضل الصادق

و ارجو ان يرجع إلينا أخانا Tyns19 ما استطاع و لو كان لديه مشاغل فقد عودنا على نشاطه و حيويته

تغريد
09-14-2009, 10:46 PM
هل لك أخي الصادق أن تحدثنا قليلا حول
كمية الحركة الزاوية
كمفهوم لأن الامور ليست واضحة تماما بالنسبة لي بهذا الخصوص

الصادق
09-15-2009, 12:15 AM
هل لك أخي الصادق أن تحدثنا قليلا حول
كمية الحركة الزاوية
كمفهوم لأن الامور ليست واضحة تماما بالنسبة لي بهذا الخصوص

مثلما ان كمية الحركة الخطية هى حاصل ضىرب الكتلة فى السرعة الخطية فان كمية الحركة الزاوية هى حاصل ضرب عزم القصور (عبارة عن ممتدد وهو يقابل كتلة القصور فى الحركة الخطية) فى السرعة الزاوية (معدل تغير الزاوية بالنسبة للزمن)

او بصيغة اخرى هو حاصل الضرب الاتجاهى لمتجه نصف قطر الدوران فى كمية الحركة الخطية
والشكل التالى يوضح دوران نقطة حول مركز ثابت وكيف يتغير اتجاه كمية الحركة الزاوية عند انعكاس اتجاه الدوران


http://upload.wikimedia.org/wikipedia/commons/0/09/Torque_animation.gif

لمزيد من المعلومات يمكنك الاطلاع على الرابط : http://en.wikipedia.org/wiki/Angular_momentum

تغريد
09-15-2009, 01:16 AM
أشكرك أخي الصادق
جزاك الله كل خير

Tyns19
09-15-2009, 03:50 AM
السلام عليكم

و ارجو ان يرجع إلينا أخانا tyns19 ما استطاع و لو كان لديه مشاغل فقد عودنا على نشاطه و حيويته
شكرا لك أخت تغريد، لقد تذكرتني تذكرك الله بكل ما هو جيد.
أنا أعتذر لكم بشدة على هذا الغياب غير المبرمج، لكنها ظروف خاصة حيث خلال أسبوع كامل لم أجلس أمام جهاز كمبيوتر الا لبضع دقائق.
لاحظت بعد هذه العودة تطورا كبيرا في الموضوع، والشكر للأخ الصادق والأخت تغريد.

Tyns19
09-15-2009, 03:52 AM
سأحاول حل التمارين المذكورة:
السؤال الأول أجابة عليه الأخت تغريد اجابة كاملة.

2-برهن ان التحويل

http://latex.codecogs.com/gif.latex?\large%20\\Q_1=\frac{1}{\ sqrt{2}}(q_1+\frac{p_1}{m\omega});% 20\qquad%20P_1=\frac{1}{\sqrt{2}}(p _1-m\omega%20q_2)%20\\%20\\%20\\Q_2=\f rac{1}{\sqrt{2}}(q_1-\frac{p_1}{m\omega});%20\qquad%20P_ 1=\frac{1}{\sqrt{2}}(p_1+m\omega%20 q_2)

(حيث m اوميقا هى ثوابت) يمثل تحويلاً قانونياً

أعتقد أن المعادلات المقصودة هي:

http://www.codecogs.com/eq.latex?\150dpi%20 \\Q_1=\frac{1}{\sqrt{2}}(q_1+\frac{ p_1}{m\omega}); \qquad P_1=\frac{1}{\sqrt{2}}(p_1-m\omega q_2) \\ \\ \\Q_2=\frac{1}{\sqrt{2}}(q_1-\frac{p_1}{m\omega}); \qquad P{\color{red} _2}=\frac{1}{\sqrt{2}}(p_1+m\omega q_2)
الان حسب الأستاذ الصادق فان التحويل القانوني يجب أن يحافظ على صيغة أقواس بواسون، وقد بين الأستاذ الصادق أن هذا مكافيء الى تحقق العلاقة التالية:

http://www.codecogs.com/eq.latex?\150dpi%20 \{Q_i,P_j\}_{q,p}=\delta_{ij}
بتطبيق العلاقة السابقة في حالتنا هذه نجد:

http://www.codecogs.com/eq.latex?\150dpi%20 \{Q_1,P_1\}_{q,p}=\frac 12\neq\delta_{11}
اذن التحويل ليس تحويل قانوني لأنه لا يحافظ على صيغة أقواس بواسون.


3- برهن ان

http://latex.codecogs.com/gif.latex?\large%20H(q+\epsilon%20\ frac{\partial%20G}{\partial%20p},p-\epsilon%20\frac{\partial%20G}{\par tial%20q},t)=H(q,p,t)+\epsilon%20\f rac{\partial%20G}{\partial%20t}

أجابت الأخت تغريد على هذا السؤال أيضا، لكن لي بعض الملاحظات أنا أتفق معكم حتى النقطة:

http://www.codecogs.com/eq.latex?\150dpi%20 \delta H =\epsilon\{H,G\}\q=\epsilon\left(\f rac{\partial G}{\partial t}-\frac{\mathrm{d}G}{\mathrm{d}t}\rig ht)


و نظرا لأن

http://latex.codecogs.com/gif.latex?\delta t=0

مما يعني أن


http://latex.codecogs.com/gif.latex?\.{G}=0

هذه العبارة محيرة جدا، أتمنى أن تعطونا تفسيرا مقنعا. كيف يمكن هذا؟؟؟

لدي تعليق اخر حول الجملة التالية:

مما يعني أنه إذا كانت الدالة المولدة للتحويل G لاتعتمد على الزمن اعتماداً صريحاً فإن الهماملتونيان تكون محفوظة
حتى اذا كان http://www.codecogs.com/eq.latex?\120dpi%20 \delta H=0 فهذا لا يعطينا الحق في القول أن الهملتونيان سيكون محفوظا وهذا لأن التغير في الهملتونيان ليس تغير كلي هذا أولا أما ثانيا فإن التغير في p و q ليس اعتباطي بل هو تغير من نوع خاص (تغير قانوني).
أضن أن الصحيح أن نقول إذا كان http://www.codecogs.com/eq.latex?\120dpi%20 \delta H=0 فان الهملتونيان يكون متماثل(ثابت) تحت التحويل القانوني.

والله أعلم.

Tyns19
09-15-2009, 03:55 AM
تمرين: اذا كانت المنظومة متماثلة تحت التحويل الدورانى, برهن ان كمية الحركة الزاوية تكون محفوظة
حسب نظرية نويثر (ان اى تماثل للمنظومة الفيزيائية تحت تحويل تولده دالة G فان هناك كمية محفوظة وهى G نفسها) وهذا هو جواب السؤال في التمرين حيث أن كمية الحركة تولد التحويل الدوراني فإن التماثل تحت هذا التحويل سيؤدي الى انحفاظ كمية الحركة الزاوية.
لكن سأجرب أن أبين هذا باستعمال النتائج التي توصل اليها الأخ الصادق.
لدينا الدالة التي تولد الدوران هي: http://www.codecogs.com/eq.latex?\150dpi%20\ G=q_1p_2-q_2p_1
لنحسب التغير في G:


http://www.codecogs.com/eq.latex?\150dpi%20 \delta G=q_1\; \delta p_2+p_2\; \delta q_1-q_2\; \delta p_1-p_1\; \delta q_2

لكن:

http://www.codecogs.com/eq.latex?\150dpi%20 \\ \delta q_1=\delta \theta \frac{\partial G}{\partial p_1}\qquad\qquad\delta q_2=\delta \theta \frac{\partial G}{\partial p_2} \\ \\ \delta p_1=-\delta \theta \frac{\partial G}{\partial q_1}\qquad\;\;\;\; \delta p_2=-\delta \theta \frac{\partial G}{\partial q_2}
إذن:

http://www.codecogs.com/eq.latex?\150dpi%20 \delta G=-\delta \theta \; q_1 \frac{\partial G}{\partial q_2}+\delta \theta\; p_2\frac{\partial G}{\partial p_1}+\delta \theta\;q_2\frac{\partial G}{\partial q_1}-\delta \theta\;p_1 \frac{\partial G}{\partial p_2}
اي:

http://www.codecogs.com/eq.latex?\150dpi%20 \delta G=-\delta \theta \left(q_1 \frac{\partial G}{\partial q_2}-p_2\frac{\partial G}{\partial p_1}-q_2\frac{\partial G}{\partial q_1}+p_1 \frac{\partial G}{\partial p_2}\right)

لكن:

http://www.codecogs.com/eq.latex?\150dpi%20 \\q_1 =\frac{\partial G}{\partial p_2}\qquad \qquad q_2=-\frac{\partial G}{\partial p_1}\\ \\ \\ p_1=-\frac{\partial G}{\partial q_2}\qquad\qquad p_2=\frac{\partial G}{\partial q_1}
إذا:


http://www.codecogs.com/eq.latex?\150dpi%20 \delta G=-\delta \theta \left(\frac{\partial G}{\partial p_2} \frac{\partial G}{\partial q_2}-\frac{\partial G}{\partial q_1} \frac{\partial G}{\partial p_1}+\frac{\partial G}{\partial p_1} \frac{\partial G}{\partial q_1}-\frac{\partial G}{\partial q_2} \frac{\partial G}{\partial p_2}\right)=0
وهو المطلوب اثباته#.

Tyns19
09-15-2009, 03:59 AM
الان لدي تساؤلات حول الموضوع
سأتحدث قليلا عن المشاركة رقم 36 الموجودة في الرابط التالي : المشاركة رقم 36 (http://www.hazemsakeek.com/vb/showthread.php?t=16467&page=4#36)
كما سأتحدث عن المشاركة رقم 47 و الموجودة على الرابط التالي: المشاركة رقم 47 (http://www.hazemsakeek.com/vb/showthread.php?t=16467&page=5#47)

1-في المشاركة رقم 36 تم تعريف التحويلات القانونية بدقة، لكن الظاهر أن هناك علاقة وثيقة بين الدالة H (الهملتونيان في النظام القديم) والدالة K (الهملتونيان في النظام الجديد). وبما أن K هي دالة بالغة الأهمية فإني أحب أن نخصص لها وقت ولو صغير وهذا مجرد اقتراح والأمر بين يديكم.

2-كما قلت فقد تم تعريف التحويلات القانونية بدقة في المشاركة رقم 36 لكن استوقفني أمر يبدو لي ذو أهمية، وهو جملة موجودة في المشاركة رقم 47 والجملة هي التالية:

التحويلات القانونية و مولدات التماثل

قلنا ان التحويلات القانونية فى ميكانيكا هملتون تُعرف بانها التحويلات من نظام الاحداثيات http://latex.codecogs.com/gif.latex?\large%20(q,p) الى نظام احداثيات جديد http://latex.codecogs.com/gif.latex?\large%20(Q,P) بحيث يحافظ التحويل على شكل معادلات هملتون. وطالما ان التحويلات القانونية تحافظ على شكل معادلات هماتون القانونية فانها ايضاً تحافظ على صيغة اقواس بوايسون

سؤالي هو هل يمكن أن نبرهن أن حفاظ التحويل على المعادلات القانونينة سيؤدي الى الحفاظ على صيغة أقواس بواسون، والسؤال بصيغة أخرى هو: هل يمكن أن نبرهن أن التحويل الذي يحافظ على شكل معادلات هاملتون (كما أتى تعريفه في المشاركة رقم 36) سيحافظ على صيغة أقواس بواسون؟؟؟ (طبعا البرهان يكون باستعمال التعريف الموجود في المشاركة 36 فقط). اذا كان هذا ممكن فأنا أتمنى أن تدرجوا البرهان لأنه كانت لي محاولات سريعة لكنها للأسف أتت عقيمة.

3-هل الحفاظ على صيغة أقواس بواسون شرط لازم حتى يكون التحويل قانوني، أم أنه شرط كافي ولازم؟؟؟؟
بصيغة أخرى، حسب كل من النعريفين الرياضين التاليين:

http://www.codecogs.com/eq.latex?\150dpi%20 \\\dot{Q}_i=\frac{\partial K}{\partial P_i}, \quad \dot{P}_i=-\frac{\partial K}{\partial Q_i} \qquad(a)\\\\\{A,B\}_{q,p}=\{A,B\}_ {Q,P}\;\;\;\qquad(b)
فالسؤال هو: هل الشرط (a) يستلزم الشرط (b)؟، أم أن الشرطان متكافئان؟؟.
هل http://www.codecogs.com/eq.latex?\110dpi%20\ (a)\Rightarrow (b) أم أن http://www.codecogs.com/eq.latex?\110dpi%20\ (a)\Leftrightarrow(b) ؟؟؟.

اسف على الاطالة، وشكرا لكم.

Tyns19
09-15-2009, 04:02 AM
مرة أخرى أعتذر لكم عن الغيبة، كما أني أعتذر عن السرعة في المشاركات الأخيرة وعدم الترتيب
وشكرا لكم على الجهود التي تبذلونها وجعلها الله في ميزان حسناتكم.

الصادق
09-15-2009, 04:05 AM
السلام عليكم اخى Tyns19
مرحباً بك لقد افتقدناك

بالنسبة لحلك للسؤال الثانى ارجو ان تنتبه لعلاقات اقواس بوايسون بين المتغيرات القديمة q و p وسوف تكتشف ان التحويل قانونى

الصادق
09-15-2009, 04:20 AM
بالنسبة لحل المسألة

http://latex.codecogs.com/gif.latex?\large%20H(q+\epsilon%20\ frac{\partial%20G}{\partial%20p},p-\epsilon%20\frac{\partial%20G}{\par tial%20q},t)=H(q,p,t)+\epsilon%20\f rac{\partial%20G}{\partial%20t}

فان مفكوك تايلور للدالة سوف يعطى البرهان بصورة مباشرة

اما حل الاخت تغريد فهو صحيح و عندنا كتبت العبارة http://latex.codecogs.com/gif.latex?\large%20\delta%20t=0

كان واضحاً بالنسبة لى انها تتحدث عن مقارنة الهملتونين مع شكل التغاير التالى

http://latex.codecogs.com/gif.latex?\large%20H(q+\delta%20q,% 20p+\delta%20p,%20t+\delta%20t)

والمقارنة تعطى

http://latex.codecogs.com/gif.latex?\large%20\delta%20q=\epsi lon%20\frac{\partial%20G}{\partial% 20p};%20\qquad%20\delta%20p=-\epsilon%20\frac{\partial%20G}{\par tial%20q};%20\qquad%20\delta%20t%20 =0

والله اعلم

الصادق
09-15-2009, 05:17 AM
لدي تعليق اخر حول الجملة التالية:


مما يعني أنه إذا كانت الدالة المولدة للتحويل G لاتعتمد على الزمن اعتماداً صريحاً فإن الهماملتونيان تكون محفوظة

حتى اذا كان http://www.codecogs.com/eq.latex?\120dpi%20 \delta H=0 فهذا لا يعطينا الحق في القول أن الهملتونيان سيكون محفوظا وهذا لأن التغير في الهملتونيان ليس تغير كلي هذا أولا أما ثانيا فإن التغير في p و q ليس اعتباطي بل هو تغير من نوع خاص (تغير قانوني).
أضن أن الصحيح أن نقول إذا كان http://www.codecogs.com/eq.latex?\120dpi%20 \delta H=0 فان الهملتونيان يكون متماثل(ثابت) تحت التحويل القانوني.

والله أعلم.

لاحظ اخى Tyns19 اننا نستطيع كتابة

http://latex.codecogs.com/gif.latex?\large%20H(q+\epsilon%20\ frac{\partial%20G}{\partial%20p},p-\epsilon%20\frac{\partial%20G}{\par tial%20q},t)=H(q,p,t)+\epsilon%20\f rac{\partial%20G}{\partial%20t}

بالصورة
http://latex.codecogs.com/gif.latex?\large%20\frac{\mathrm{d} G}{\mathrm{d}%20t}=\frac{\partial%2 0G}{\partial%20t}+\{G,H\}=0

وهكذا اذا كانت G لا تعتمد على الزمن اعتماداً صريحاَ فان الدالة المولدة سوف تكون محفوظة لان http://latex.codecogs.com/gif.latex?\large%20\frac{\mathrm{d} G}{\mathrm{d}%20t}%20=0
وطالما ان G تتبادل مع الهملتونيان عن طريق اقواس بوايسون http://latex.codecogs.com/gif.latex?\large%20\{G,H\}=0
فان الهملتونيان يكون ايضاً محفوظاً
وهذا ماتوصلت له اختى الكريمة تغريد فى حلها.

والله اعلم

لم افهم قولك فى العبارة


ثانيا فإن التغير في p و q ليس اعتباطي بل هو تغير من نوع خاص (تغير قانوني).
هل كنت تتحدث عن التغير ام عن التحويل ؟ لان التغيرات فى p وq هنا اعتباطية (ويجب ان تكون متناهية الصغر فى حالة التحويلات القانونية متناهية الصغر) اما التحويل الذى تولده G فهو تحويل قانونى يُستلزم ان يحافظ على البناء القانونى

والله اعلم

الصادق
09-15-2009, 05:26 AM
حسب نظرية نويثر (ان اى تماثل للمنظومة الفيزيائية تحت تحويل تولده دالة G فان هناك كمية محفوظة وهى G نفسها) وهذا هو جواب السؤال في التمرين حيث أن كمية الحركة تولد التحويل الدوراني فإن التماثل تحت هذا التحويل سيؤدي الى انحفاظ كمية الحركة الزاوية.
لكن سأجرب أن أبين هذا باستعمال النتائج التي توصل اليها الأخ الصادق.
لدينا الدالة التي تولد الدوران هي: http://www.codecogs.com/eq.latex?\150dpi%20\ G=q_1p_2-q_2p_1
لنحسب التغير في G:


http://www.codecogs.com/eq.latex?\150dpi%20 \delta G=q_1\; \delta p_2+p_2\; \delta q_1-q_2\; \delta p_1-p_1\; \delta q_2

لكن:

http://www.codecogs.com/eq.latex?\150dpi%20 \\ \delta q_1=\delta \theta \frac{\partial G}{\partial p_1}\qquad\qquad\delta q_2=\delta \theta \frac{\partial G}{\partial p_2} \\ \\ \delta p_1=-\delta \theta \frac{\partial G}{\partial q_1}\qquad\;\;\;\; \delta p_2=-\delta \theta \frac{\partial G}{\partial q_2}
إذن:

http://www.codecogs.com/eq.latex?\150dpi%20 \delta G=-\delta \theta \; q_1 \frac{\partial G}{\partial q_2}+\delta \theta\; p_2\frac{\partial G}{\partial p_1}+\delta \theta\;q_2\frac{\partial G}{\partial q_1}-\delta \theta\;p_1 \frac{\partial G}{\partial p_2}
اي:

http://www.codecogs.com/eq.latex?\150dpi%20 \delta G=-\delta \theta \left(q_1 \frac{\partial G}{\partial q_2}-p_2\frac{\partial G}{\partial p_1}-q_2\frac{\partial G}{\partial q_1}+p_1 \frac{\partial G}{\partial p_2}\right)

لكن:

http://www.codecogs.com/eq.latex?\150dpi%20 \\q_1 =\frac{\partial G}{\partial p_2}\qquad \qquad q_2=-\frac{\partial G}{\partial p_1}\\ \\ \\ p_1=-\frac{\partial G}{\partial q_2}\qquad\qquad p_2=\frac{\partial G}{\partial q_1}
إذا:


http://www.codecogs.com/eq.latex?\150dpi%20 \delta G=-\delta \theta \left(\frac{\partial G}{\partial p_2} \frac{\partial G}{\partial q_2}-\frac{\partial G}{\partial q_1} \frac{\partial G}{\partial p_1}+\frac{\partial G}{\partial p_1} \frac{\partial G}{\partial q_1}-\frac{\partial G}{\partial q_2} \frac{\partial G}{\partial p_2}\right)=0
وهو المطلوب اثباته#.


حل صحيح و جميل جداً. جزاك اله خيراً اخى Tyns19 وذادك علماً و حكمة

ملاحظة:
يمكن ايضاً ان نصل لنفس هذه النتيجة مباشرةً عن طريق استخدام اقواس بوايسون

Tyns19
09-15-2009, 05:33 AM
السلام عليكم اخى Tyns19
مرحباً بك لقد افتقدناك
وعليك السلام أخي الصادق، أنا أيضا افتقدتكم كثيرا.

بالنسبة لحلك للسؤال الثانى ارجو ان تنتبه لعلاقات اقواس بوايسون بين المتغيرات القديمة q و p وسوف تكتشف ان التحويل قانونى
سأدرج حلي مفصلا وأتمنى أن لا أكون قد ارتكبت خطأ فادح، أولا التحويلات المعطات هي:

http://www.codecogs.com/eq.latex?\120dpi%20 \\Q_1=\frac{1}{\sqrt{2}}(q_1+\frac{ p_1}{m\omega}); \qquad P_1=\frac{1}{\sqrt{2}}(p_1-m\omega q_2) \\ \\ \\Q_2=\frac{1}{\sqrt{2}}(q_1-\frac{p_1}{m\omega}); \qquad P{\color{red} _2}=\frac{1}{\sqrt{2}}(p_1+m\omega q_2)
(لاحظ رقم 2 المكتوب بالأحمر أنا أعتقد وجود خطأ كتابي في املاء التمرين).
الان حسب تعريف أقواس بواسون فإن:

http://www.codecogs.com/eq.latex?\120dpi%20 \{Q_1,P_1\}_{q,p}=\frac{\partial Q_1}{\partial q_1}\frac{\partial P_1}{\partial p_1}-\frac{\partial Q_1}{\partial p_1}\frac{\partial P_1}{\partial q_1}+\frac{\partial Q_1}{\partial q_2}\frac{\partial P_1}{\partial p_2}-\frac{\partial Q_1}{\partial p_2}\frac{\partial P_1}{\partial q_2}
لكن:

http://www.codecogs.com/eq.latex?\120dpi%20 \\\frac{\partial Q_1}{\partial q_1}=\frac 1{\sqrt{2}};\;\frac{\partial P_1}{\partial p_1}=\frac 1{\sqrt{2}};\;\frac{\partial Q_1}{\partial p_1}=\frac 1{\sqrt{2}\; m\omega};\;\frac{\partial P_1}{\partial q_1}=0\\\\\frac{\partial Q_1}{\partial q_2}=0;\;\frac{\partial P_1}{\partial p_2}=0;\;\frac{\partial Q_1}{\partial p_2}=0;\;\frac{\partial P_1}{\partial q_2}=-\frac {m\omega}{\sqrt{2}}
وهكذا فان الحدود التي تبقى هي:

http://www.codecogs.com/eq.latex?\120dpi%20 \{Q_1,P_1\}_{q,p}=\frac{\partial Q_1}{\partial q_1}\frac{\partial P_1}{\partial p_1}=\frac 1{\sqrt{2}}\cdot\frac 1{\sqrt{2}}=\frac12
يبدو لي الحل صحيح، اذا ارتأيتم غير هذا أرجو توضيح القطعة الموجود فيها الخطأ.

الصادق
09-15-2009, 06:13 AM
نعم هناك خطاء املائى فى طباعة المعادلات لم انتبه له, ارجو من جميع الاخوة التدقيق دائماً فى المعادلات لان اخطاء الطباعة غير المقصودة ورادة خصوصاً عند كتابة عدد كبير من المعادلات

المعادلات الصحيحة هى

http://latex.codecogs.com/gif.latex?\large%20\\Q_1=\frac{1}{\ sqrt{2}}(q_1+\frac{p_2}{m\omega});% 20\qquad%20P_1=\frac{1}{\sqrt{2}}(p _1-m\omega%20q_2)%20\\%20\\%20\\Q_2=\f rac{1}{\sqrt{2}}(q_1-\frac{p_2}{m\omega});%20\qquad%20P_ 2=\frac{1}{\sqrt{2}}(p_1+m\omega%20 q_2)

شكراً لك اخى على التنبيه

تغريد
09-15-2009, 04:04 PM
مرحبا بعودتك أخي Tyns19 فقد افتدقناك و قد كان تواجدك في الموضوع و لا يزال يشكل مع المجهود المشكور الذي يقوم به أخي الصادق ثنائي رائع ، حفظكم و وفقكم الله

بالنسبة لقولك
"في المشاركة رقم 36 تم تعريف التحويلات القانونية بدقة، لكن الظاهر أن هناك علاقة وثيقة بين الدالة H (الهملتونيان في النظام القديم) والدالة K (الهملتونيان في النظام الجديد). وبما أن K هي دالة بالغة الأهمية فإني أحب أن نخصص لها وقت ولو صغير وهذا مجرد اقتراح والأمر بين يديكم."


ربما يكون سؤالي في ذات الإطار

أخي الصادق : لقد وجدنا أن الدالة K هي نفسها الدالة H في كل الأحوال ، فهل يجب أن تكون الدالة K تعبر عن طاقة النظام الكلية تحت أي تحويل قانوني

أعتقد أن الإجابة يفترض أن تكون نعم فطالما معادلات هاملتون للحركة لم تتغير يمكننا القول أن هذا يعني أن الطاقة محفوظة؟


و لكن

بحسب تطور الأسئلة أعتقد أنه في حال أن الدالة H و من ثم الدالة G تعتمد اعتمادا صريحا على الزمن سوف تختلف K عن H و لكن في هذه الحالة كيف سيكون الوضع و هل سيظل التحويل قانوني

مثلا الحركة تحت تأثير قوى تزايدية ثابته في حال وجود قوى احتكاك

أرجو أن يكون سؤالي ذا معنى و إلا لا تزعج نفسك بالرد أخي الصادق

Tyns19
09-22-2009, 02:52 AM
السلام عليكم، وعيد مبارك
لقد حدث لي التباس في حل الأخت تغريد للسؤال الثالث ولقد سبق وأن بينت النقطة المقصودة، لكن سأتكلم عنها ببعض من التفصيل الان

لقد تمثل الالتباس في ثلاث نقاط أساسية هي كالتالي:

النقطة الأولى:
البرهان على أن:

http://www.codecogs.com/eq.latex?\150dpi%20\ H(q+\epsilon \frac{\partial G}{\partial p},p-\epsilon \frac{\partial G}{\partial q},t)=H(q,p,t)+\epsilon \frac{\partial G}{\partial t}

ولتوضيح نقطة الاختلاف فأنا أتفق معكم حتى النقطة التالية:

http://www.codecogs.com/eq.latex?\140dpi%20\ H(q+\epsilon \frac{\partial G}{\partial p},p-\epsilon \frac{\partial G}{\partial q},t)=H(q,p,t)+\epsilon\{H,G\}_q=H( q,p,t)+\epsilon\left(\frac{\partial G}{\partial t}-\frac{\mathrm{d}G}{\mathrm{d}t}\rig ht)
أما الباقي فأنا أرجو توضيحه ببعض التروي.

النقطة الثانية:
وهي حول تعليقي حول هذه العبارة:

و نظرا لأن

http://latex.codecogs.com/gif.latex?\delta t=0

مما يعني أن


http://latex.codecogs.com/gif.latex?\.{G}=0

لقد جاءت صياغة هذه العبارة بمفهوم (بما إن فإن) والفهوم الذي فهمته من هذه العبارة يمكن تلخيصه في الشكل الرياضي التالي:

http://www.codecogs.com/eq.latex?\150dpi%20 \delta t=0\Rightarrow \dot{G}=0
فهل هذا هو المقصود؟؟.

النقطة الثالثة:
يمكن صياغة الاختلاف في هذه النقطة بالشكل التالي:
اذا كان:

http://www.codecogs.com/eq.latex?\150dpi%20\ H(q+\epsilon \frac{\partial G}{\partial p},p-\epsilon \frac{\partial G}{\partial q},t)=H(q,p,t)
فهل هذا يعني أن الهملتونيان كمية محفوظة؟
كانت اجابتك أخي الصادق أنت والأخت تغريد (نعم)، وكانت اجابتي (لا) وأنا لازلت أحتفظ باجابتي، كما أعتقد أنكم أيضا لا تزالون تحتفظون باجابتكم والاختلاف ليس شيئا جيدا، وأنا أكيد اننا سنتفق على هذه النقطة جيدا بعد المناقشة لكنني لا أريد مناقشتها (النقطة الثالثة) حاليا، فربما تتضح الأمور بعد جوابكم على النقطتين الأولى والثانية
وسيكون لنا حديث حول النقطة الثالثة فيما بعد انشاء الله.

وشكر لكم.

الصادق
09-28-2009, 09:07 PM
السلام عليكم، وعيد مبارك
لقد حدث لي التباس في حل الأخت تغريد للسؤال الثالث ولقد سبق وأن بينت النقطة المقصودة، لكن سأتكلم عنها ببعض من التفصيل الان

لقد تمثل الالتباس في ثلاث نقاط أساسية هي كالتالي:

النقطة الأولى:
البرهان على أن:

http://www.codecogs.com/eq.latex?\150dpi%20\ H(q+\epsilon \frac{\partial G}{\partial p},p-\epsilon \frac{\partial G}{\partial q},t)=H(q,p,t)+\epsilon \frac{\partial G}{\partial t}

ولتوضيح نقطة الاختلاف فأنا أتفق معكم حتى النقطة التالية:

http://www.codecogs.com/eq.latex?\140dpi%20\ H(q+\epsilon \frac{\partial G}{\partial p},p-\epsilon \frac{\partial G}{\partial q},t)=H(q,p,t)+\epsilon\{H,G\}_q=H( q,p,t)+\epsilon\left(\frac{\partial G}{\partial t}-\frac{\mathrm{d}G}{\mathrm{d}t}\rig ht)
أما الباقي فأنا أرجو توضيحه ببعض التروي.

النقطة الثانية:
وهي حول تعليقي حول هذه العبارة:

لقد جاءت صياغة هذه العبارة بمفهوم (بما إن فإن) والفهوم الذي فهمته من هذه العبارة يمكن تلخيصه في الشكل الرياضي التالي:

http://www.codecogs.com/eq.latex?\150dpi%20 \delta t=0\Rightarrow \dot{G}=0
فهل هذا هو المقصود؟؟.

النقطة الثالثة:
يمكن صياغة الاختلاف في هذه النقطة بالشكل التالي:
اذا كان:

http://www.codecogs.com/eq.latex?\150dpi%20\ H(q+\epsilon \frac{\partial G}{\partial p},p-\epsilon \frac{\partial G}{\partial q},t)=H(q,p,t)
فهل هذا يعني أن الهملتونيان كمية محفوظة؟
كانت اجابتك أخي الصادق أنت والأخت تغريد (نعم)، وكانت اجابتي (لا) وأنا لازلت أحتفظ باجابتي، كما أعتقد أنكم أيضا لا تزالون تحتفظون باجابتكم والاختلاف ليس شيئا جيدا، وأنا أكيد اننا سنتفق على هذه النقطة جيدا بعد المناقشة لكنني لا أريد مناقشتها (النقطة الثالثة) حاليا، فربما تتضح الأمور بعد جوابكم على النقطتين الأولى والثانية
وسيكون لنا حديث حول النقطة الثالثة فيما بعد انشاء الله.

وشكر لكم.

السلام عليكم اخى Tyns19 وكل عام وانتم بخير
اعتزر لك اخى Tyns19 عنى وعن اختى الكريمة تغريد فقد انشغلنا فى الفترة السابقة فى مشروع رياضى فى منتدى اخر

اعتقد ان الالتباس قد نجم من عدم ملاحظة ان الدالة المولدة للتحويل القانونى هى ثابت من ثوابت الحركة
انظر نظرية نوزر فى المشاركة رقم 52


ان اى تماثل للمنظومة الفيزيائية تحت تحويل تولده دالة G فان هناك كمية محفوظة وهى G نفسها

الان دعنا نرجع الى المسألة
نعلم ان مفكوك تايلور لاى دالة http://latex.codecogs.com/gif.latex?\large%20F(x+\delta%20x,y +\delta%20y,z+\delta%20z) يُعطى ب
http://latex.codecogs.com/gif.latex?\large%20F(x+\delta%20x,y +\delta%20y,z+\delta%20z)=F(x,y,z)+ \delta%20x%20\frac{\partial%20F}{\p artial%20x}+\delta%20y%20\frac{\par tial%20F}{\partial%20y}+\delta%20z\ frac{\partial%20F}{\partial%20z}+\m athcal{O}(\delta^2)

وهكذا فان
http://latex.codecogs.com/gif.latex?\large%20H(q+\delta%20q,p +\delta%20p,%20t+\delta%20t)=H(q,p, t)+\delta%20q%20\frac{\partial%20H} {\partial%20q}+\delta%20p%20\frac{\ partial%20H}{\partial%20p}+\delta%2 0t\frac{\partial%20H}{\partial%20t}

ولكن واضح ان التغيرات المعطاة فى المسألة هى
http://latex.codecogs.com/gif.latex?\large%20\delta%20q=\epsi lon%20\frac{\partial%20G}{\partial% 20p};%20\qquad%20\delta%20p=-\epsilon%20\frac{\partial%20G}{\par tial%20q};\qquad%20\delta%20t%20=0

وبالتعويض المباشر نحصل على

http://latex.codecogs.com/gif.latex?\large%20H\left(q+\epsilo n%20\frac{\partial%20H}{\partial%20 p},p-\epsilon%20\frac{\partial%20H}{\par tial%20q},%20t+0\right)=H(q,p,t)+%2 0\epsilon%20\frac{\partial%20G}{\pa rtial%20p}%20\frac{\partial%20H}{\p artial%20q}-\epsilon\frac{\partial%20G}{\partia l%20q}%20\frac{\partial%20H}{\parti al%20q}+0

واذا استخدمنا معادلات هملتون القانونية
http://latex.codecogs.com/gif.latex?\large%20H\left(q+\epsilo n%20\frac{\partial%20H}{\partial%20 p},p-\epsilon%20\frac{\partial%20H}{\par tial%20q},%20t\right)=H(q,p,t)-%20\epsilon\left(%20\frac{\partial% 20G}{\partial%20p}\dot{p}+\frac{\pa rtial%20G}{\partial%20q}%20\dot{q}\ right%20)


اخيراً بالاستفادة من ان الدالة المولدة للتحويل القانونى ثابت من ثوابت الحركة

http://latex.codecogs.com/gif.latex?\large%200=\frac{\mathrm{ d}G%20}{\mathrm{d}%20t}=\frac{\part ial%20G}{\partial%20q}\dot{q}+\frac {\partial%20G}{\partial%20p}\dot{p} +\frac{\partial%20G}{\partial%20t}= 0\Rightarrow%20\frac{\partial%20G}{ \partial%20q}\dot{q}+\frac{\partial %20G}{\partial%20p}\dot{p}=-\frac{\partial%20G}{\partial%20t}

وبالتعويض فى المعادلة اعلاه نحصل على البرهان المطلوب

http://latex.codecogs.com/gif.latex?\large%20H(q+\epsilon%20\ frac{\partial%20H}{\partial%20p},p-\epsilon%20\frac{\partial%20H}{\par tial%20q},t)=H(q,p,t)+\epsilon%20\f rac{\partial%20G}{\partial%20t}

اما عن بقية المناقسة اقتبس لك ما جاء فى المشاركة رقم 71


لاحظ اخى Tyns19 اننا نستطيع كتابة

http://latex.codecogs.com/gif.latex?\large%20H(q+\epsilon%20\ frac{\partial%20G}{\partial%20p},p-\epsilon%20\frac{\partial%20G}{\par tial%20q},t)=H(q,p,t)+\epsilon%20\f rac{\partial%20G}{\partial%20t}

بالصورة
http://latex.codecogs.com/gif.latex?\large%20\frac{\mathrm{d} G}{\mathrm{d}%20t}=\frac{\partial%2 0G}{\partial%20t}+\{G,H\}=0

وهكذا اذا كانت G لا تعتمد على الزمن اعتماداً صريحاَ فان الدالة المولدة سوف تكون محفوظة لان http://latex.codecogs.com/gif.latex?\large%20\frac{\mathrm{d} G}{\mathrm{d}%20t}%20=0
وطالما ان G تتبادل مع الهملتونيان عن طريق اقواس بوايسون http://latex.codecogs.com/gif.latex?\large%20\{G,H\}=0
فان الهملتونيان يكون ايضاً محفوظاً
وهذا ماتوصلت له اختى الكريمة تغريد فى حلها.

والله اعلم


والله اعلم





والله اعلم

للمعرفة
09-28-2009, 09:18 PM
شكرا لك اخى على الموضوع الرائع

الصادق
09-28-2009, 09:42 PM
بارك الله فيك اختى الكريمة للمعرفة

وحش النسبية
06-09-2012, 08:57 PM
موضوع يستحق الرفع للأهمية

انشتاين فرداي
03-25-2013, 12:36 AM
نموذج القشرة (****l model):
ان نموذج القشرة يفترض بان النيوكليونات تتاثر بمجال يخص النواة ككل بدلاً من مجال النيوكليونات المنفردة، وان حركة كل نيوكليون لا تعتمد على حركة اي نيوكليون اخر ويخضع لمبدأ باولي للانفراد(الاستبعاد).
ان النوى التي تمتلك عددا من البروتونات او النيوترونات مساوياً للاعداد السحرية (2,8,20,28,50,82,126) نجدها اكثر وفرةً واكثر استقراراً من النوى الاخرى.
ان نموذج القشرة للنواة وجد لتفسير الاعداد السحرية والصفات النووية الاخرى، ودالة الطاقة الكامنه لهذا النموذج تشبه منخفض جهد مربع لكن جوانبه مستديرة.
لكي نحصل على مستويات طاقة تتفق مع وجود الاعداد السحرية، افترض كل من ماير و جينسون وجود ازدواجاً بين الزخم الزاوي المداري والبرم الذاتي للنيوكليون او ما يسمى ازدواج (L.S)

ان نموذج القشرة يستطيع ان يستنتج قيمة الزخم الزاوي الكلي للنواة والتماثل بالاستناد الى القواعد التالية:
1- يكون الزخم الزاوي الكلي للحالة الارضية(المستوي الارضي) لنواة من النوع (زوجية-زوجية) مساوياً الى الصفر والتماثل هو موجب
j=0+
الاشارة (+) تعني ان التماثل (?) هو موجب
2- ان النواة من النوع (زوجية-فردية) او (فردية-زوجية) يكون j للحالة الارضية (المستوي الارضي) للنيوكليون المنفرد فقط، وقيمته عدد نصف كامل ويحسب التماثل (?) من
?=(-1)l
3- النواة من النوع (فردية فردية) نحسب قيمة j لها للحالة الارضية كما ياتي
j=jn+jp
حيث ان jn: تمثل الزخم للنيوترون
jp : تمثل الزخم للبروتون
ويكون الزخم الزاوي الكلي عدد صحيح محصور بين
| jn- jp| ? j ? jn+jp
اما التماثل فيحسب من
?=(-1) ln+lp